Comprehensive Review in Clinical Neurology: A Multiple Choice Question Book for the Wards and Boards

Chapter 6. Movement Disorders

Question

Questions 1–2

1. Regarding the anatomy of the basal ganglia, which of the following statements is incorrect?

a. The striatum consists of the caudate and putamen

b. The lenticular nucleus consists of the putamen and globus pallidus

c. The majority of cortical projections to the basal ganglia are to the striatum and subthalamic nucleus

d. The major outflow of the basal ganglia arises in the putamen

e. The globus pallidus projects to the thalamus, which, in turn, projects to multiple cortical areas

2. Regarding basal ganglia circuits, which of the following statements is correct?

a. The indirect pathway is excitatory: by increasing inhibition of the globus pallidus interna (GPi), it increases thalamic output

b. The direct pathway is inhibitory: by increasing inhibition of the GPi, it decreases thalamic output

c. Projections from the GPi to the thalamus are inhibitory

d. Projections of the thalamus to the cortex are inhibitory

e. Projections from the subthalamic nucleus to the GPi are inhibitory

Questions 3–4

3. Which of the following statements is correct regarding the direct and indirect circuits of the basal ganglia?

a. Hyperkinetic movement disorders result from increased activity in the indirect pathway

b. Hypokinetic movement disorders result from increased activity in the direct pathway

c. The net effect of activity in the indirect pathway is to normally increase movement

d. The net effect of activity in the direct pathway is to normally inhibit movement

e. Hyperkinetic movement disorders result from reduced activity in the indirect pathway

4. Which of the following statements is incorrect?

a. The substantia nigra pars compacta (SNc) normally inhibits the indirect pathway and excites the direct pathway

b. Parkinson’s disease (PD) results from degeneration of the SNc, which leads to reduced activity in the direct pathway and increased activity in the indirect pathway, leading to bradykinesia

c. In Huntington’s disease, a reduction of activity in the indirect pathway disinhibits the thalamus, leading to increased activity, in the form of chorea and other extraneous movements

d. Abnormalities in the direct and indirect pathway do not account for all of the features of most movement disorders such as PD, in which both hyperkinetic and bradykinetic movements occur in combination

e. Decreased thalamocortical activity results in hyperkinetic movement disorders and increased thalamocortical activity results in hypokinetic movement disorders

5. Regarding neurotransmission in the basal ganglia, which of the following statements is incorrect?

a. Glutamate is the major excitatory neurotransmitter of the basal ganglia

b. GABA is the major inhibitory neurotransmitter of the basal ganglia

c. GABA is the major neurotransmitter released in striatal projections to the globus pallidus and in globus pallidus interna projections to the thalamus

d. D1 receptors are primarily found on neurons involved in the indirect pathway, and D2 receptors on neurons involved in the direct pathway

e. The substantia nigra pars compacta inhibits the indirect pathway via activity at D2 receptors and excites the direct pathway via activity at D1 receptors

Questions 6–7

6. A 52-year-old man presented to the clinic complaining of a tremor. His wife had noticed a tremor over the prior year, mainly while he was at rest, such as while watching television. He also complained of walking slower, and his wife reported that his voice was softer. On directed questioning, his handwriting had gotten smaller as well. He was otherwise healthy, and had no complaints of cognitive problems. On examination, he had a moderate frequency rest tremor in the right hand, with mild cogwheeling in the right arm. On gait examination, he had a slightly flexed posture and he swung his right arm less than the left. Finger tapping was slower on the right. Examination was otherwise unremarkable. What is the most likely diagnosis in this patient?

a. Idiopathic Parkinson’s disease

b. DLB

c. Vascular parkinsonism

d. Drug-induced parkinsonism

e. Corticobasal ganglionic degeneration

7. A 63-year-old woman presented to the clinic complaining of walking difficulties of 2 years’ duration that had worsened over time. She had first noticed less agility of her right hand and felt her right foot was dragging slightly. She had always been a fast walker, but more recently, she had noticed that she had significantly slowed down. She also complained that she had a hard time projecting her voice, and her handwriting had gotten smaller. She denied having tremor. On examination, she had reduced facial expression, reduced blinking frequency, and moderate cogwheeling rigidity of the right upper extremities. There was mild cogwheeling in the left upper extremity with facilitation. On gait examination, she had a stooped posture, and reduced arm swing bilaterally, right more than left. She turned en bloc. On the pull test (pulled backward by examiner while stationary), she was able to take one step back and prevent herself from falling. No tremor was evident on examination. Finger tapping was slow bilaterally, right more than left. What is the most likely diagnosis in this patient?

a. Idiopathic Parkinson’s disease

b. DLB

c. Vascular parkinsonism

d. Drug-induced parkinsonism

e. Progressive supranuclear palsy

8. Regarding the clinical features of idiopathic Parkinson’s disease (PD), which of the following statements is incorrect?

a. The tremor is typically a resting tremor of moderate frequency, 4 to 6 Hz

b. Postural tremor can occur; with assumption of a specific posture, there is usually a latency of a few seconds before the tremor reemerges

c. Loss of postural reflexes does not occur in idiopathic PD; its presence suggests an alternative diagnosis

d. Freezing is most often a feature of advanced disease

e. The rigidity is often cogwheeling, and can be proximal or distal, involving both axial and limb muscles

9. Which of the following statements is correct regarding the nonmotor symptoms seen in patients with idiopathic Parkinson’s disease (PD)?

a. Shoulder pain in a patient with PD should prompt an extensive work-up for orthopedic problems, as the pain would not be attributable to the PD itself

b. Urinary frequency, urgency, and nocturia occur only in male PD patients with prostatic hypertrophy

c. In a patient with parkinsonism and orthostatic hypotension, the diagnosis cannot be idiopathic PD; it must be multiple-system atrophy

d. Constipation is a prominent symptom in patients with PD, and may predate the motor symptoms for years

e. REM sleep behavioral disorder in a patient with parkinsonism implies the diagnosis is DLB, not idiopathic PD

10. A 63-year-old woman with a 5-year history of idiopathic Parkinson’s disease (PD) presents to her neurologist for routine follow-up. Her motor examination is stable, and she feels that her regimen of dopaminergic therapy is adequate. However, her affect is blunted and she describes multiple complaints, including loss of interest in social activities, reduced appetite, increased sleep, and a sense of hopelessness. Which of the following statements is correct in relation to this patient?

a. She is suffering from depression, and should be treated with appropriate pharmacologic and nonpharmacologic measures targeted at the depression specifically

b. These symptoms are part of her PD and are best treated by optimizing her dopaminergic regimen

c. She is suffering from an adjustment disorder due to the diagnosis of PD, and just needs some encouragement

d. Her affect is blunted only because of the hypomimia that occurs in PD

e. Her symptoms are likely due to the sedating side effects of her dopaminergic therapy, and her dopaminergic therapy regimen should be reduced

11. A 72-year-old man diagnosed with idiopathic Parkinson’s disease (PD) 15 years earlier presented for follow-up with his neurologist. He had bilateral asymmetric tremor, rigidity, and bradykinesia, all of which had worsened. He had been falling frequently recently, and had started using a walker. The patient’s wife reported concerns about his memory; he was forgetting even basic things like his son’s phone number, and he had forgotten his car engine turned on overnight in recent weeks. He was also somewhat more withdrawn and less sociable. The patient denied memory loss. He was referred for formal neuropsychologic evaluation, which showed markedly impaired memory and impaired visuospatial abilities. Word fluency was moderately impaired. There had been significant declines in performance as compared to neuropsychologic evaluation done 10 years prior; on that evaluation, the patient had normal or borderline impaired scores. What is the most likely diagnosis in this patient?

a. FTD occurring concomitant with idiopathic PD

b. He was misdiagnosed 15 years earlier; he has had DLB all along

c. PD with dementia

d. Normal-pressure hydrocephalus

e. He was misdiagnosed 15 years earlier; he has corticobasal ganglionic degeneration

12. Which of the following statements is incorrect regarding the pathophysiology of idiopathic Parkinson’s disease (PD)?

a. Results from degeneration of dopaminergic neurons in the substantia nigra pars reticulata

b. Pathologic changes occur in the olfactory regions, dorsal motor nucleus of the vagus, brain stem (locus coeruleus and raphe nuclei), and basal ganglia, as well as enteric nervous system

c. A subset of patients have familial, monogenetic forms of PD

d. The pathologic hallmark of idiopathic PD is deposition of Lewy bodies

e. Idiopathic PD is a synucleinopathy

13. Which of the following Parkinson’s disease medication-mechanism of action pairs is incorrect?

a. Carbidopa–dopa-decarboxylase inhibitor

b. Ropinirole–agonist at D2 and D3 receptors

c. Rasagiline–monoamine oxidase type B inhibitor

d. Entacapone–nonspecific monoamine oxidase inhibitor

e. Trihexyphenidyl–anticholinergic

14. Which of the following statements regarding side effects of therapies for idiopathic Parkinson’s disease is incorrect?

a. Anticholinergic agents can cause significant cognitive dysfunction

b. Dopamine agonists carry little risk of impulse control problems unless the patient has a prior history of impulse control problems

c. Levodopa can cause extraneous movements, dyskinesias, and the risk increases with increasing dose and duration of therapy

d. One of the metabolites of selegiline is methamphetamine, and side effects from it include insomnia

e. Sedation is a prominent side effect of dopamine agonists

15. A 79-year-old male presents to the clinic complaining of tremor. On examination, he has moderate bradykinesia, and bilateral tremor and rigidity, both worse in the right arm. His wife reports that he takes several naps during the day and that he is sometimes forgetful. The wife and the patient minimized his cognitive symptoms. What is the most appropriate treatment for this patient?

a. Ropinirole

b. Carbidopa-levodopa

c. Pramipexole

d. Trihexyphenidyl

e. Bromocriptine

16. Regarding the surgical treatment of idiopathic Parkinson’s disease (PD), which of the following statements is incorrect?

a. If the motor symptoms are markedly asymmetric, with the less affected side being minimally involved, unilateral deep brain stimulation (DBS) can be done

b. DBS is effective in treating tremor and bradykinesia

c. DBS is effective in treating gait freezing and falls

d. Significant cognitive impairment is a contraindication to DBS

e. DBS targets for PD have included the subthalamic nucleus, globus pallidus interna, and the ventral intermediate nucleus of the thalamus

17. A 64-year-old woman presented to the clinic with a 2-year history of falling that had progressed over time. She reported she would just suddenly fall, without a trigger. She noticed difficulty going downstairs. On examination, in primary gaze, she had subtle jerk nystagmus. Her neck was hyperextended and she had difficulty flexing it. She had impaired vertical gaze, predominantly on downward gaze. However, on vertical oculocephalic maneuver, downward gaze was normal. There was mild bilateral rigidity in the upper extremities. On the pull test (examiner pulls the patient backward, instructing the patient to take a step back and prevent falling), she had significant retropulsion and would have fallen if not caught by the examiner. What is the most likely diagnosis in this patient?

a. Idiopathic Parkinson’s disease with early falling

b. Multiple-system atrophy

c. Progressive supranuclear palsy

d. Normal-pressure hydrocephalus

e. Corticobasal ganglionic degeneration

Questions 18–19

18. A 54-year-old woman presents with a 1-year history of light-headedness when rising from a seated position. This had worsened over time, and she uses a wheelchair to get around for fear of having a syncopal episode. On directed questioning, she reports urinary urgency for several years, with urge incontinence for the past 6 months. On examination in the clinic, her sitting blood pressure is 120/80 mm Hg. Standing blood pressure is 80/60 mm Hg, associated with significant light-headedness. Examination also shows an involuntary flexed posture of her neck, bilateral rigidity in the upper and lower extremities, and a bilateral slight rest tremor of moderate frequency. What is the most likely diagnosis in this patient?

a. Idiopathic Parkinson’s disease with associated dysautonomia

b. Multiple-system atrophy

c. Progressive supranuclear palsy

d. Primary autonomic failure

e. Corticobasal ganglionic degeneration

19. A 56-year-old man presented to the clinic with bilateral upper extremity rest tremor of 6 months’ duration. On examination, he had bilateral moderate-frequency rest tremor, slightly worse in the right upper extremity compared to the left, and bilateral cogwheel rigidity that appeared to be equal in both extremities. He had significant bradykinesia affecting both sides of his body. He was prescribed levodopa, which had little effect on his symptoms. On follow-up 1 year later, he reported significant gait instability. Examination showed bilateral dysmetria and his gait was wide-based and slightly lurching. MRI is shown in Figure 6.1. What is the most likely diagnosis in this patient?

FIGURE 6.1 Axial T2-weighted MRI

a. Idiopathic Parkinson’s disease

b. Multiple-system atrophy

c. Progressive supranuclear palsy

d. Primary autonomic failure

e. Corticobasal ganglionic degeneration

20. A 62-year-old male presented to the clinic complaining of arm pain. He reported that his right arm felt stiff and painful, and it would tremor at rest. He also reported difficulty walking, but had not fallen. On examination, he had dystonia of the right arm with flexed posture at the elbow and wrist. He had occasional whole-body jerks. Sensation to light-touch, pinprick, and other primary sensory modalities was normal, but with his eyes closed, he could not identify objects placed in his right hand or numbers drawn on his palm. What is the most likely diagnosis in this patient?

a. Idiopathic Parkinson’s disease

b. Multiple-system atrophy

c. Progressive supranuclear palsy

d. Normal-pressure hydrocephalus

e. Corticobasal ganglionic degeneration

21. Which of the following statements regarding secondary parkinsonism is incorrect?

a. Postencephalitic parkinsonism has been seen following infection with influenza virus, West Nile virus, and Japanese encephalitis virus

b. Vascular parkinsonism classically affects the lower extremities more than the upper extremities, and rigidity and bradykinesia predominate the picture, with little tremor

c. Drug-induced parkinsonism is most commonly seen with antipsychotic agents

d. Magnesium toxicity can lead to parkinsonism

e. Parkinsonism can be a feature of normal-pressure hydrocephalus and other causes of chronic hydrocephalus

Questions 22–24

22. A 33-year-old overweight woman with asthma presents to the clinic complaining of tremor. She reports she had a tremor since her teenage years but it had not really bothered her until the prior year. Her handwriting was starting to be “shaky” and she was not able to eat soup without spilling. She worked in construction and often operated heavy machinery, and was concerned that her tremor was putting herself and others in danger. Her father and paternal grandfather had a similar tremor. On examination, there was no tremor at rest. With outstretched arms, or while pouring water from one cup to another, a prominent bilateral high-frequency tremor was observed. What is the most likely diagnosis in this patient?

a. Enhanced physiologic tremor

b. Essential tremor

c. Dystonic tremor

d. Task-specific tremor

e. Rubral tremor

23. Regarding the disorder depicted in question 22, which of the following statements is incorrect?

a. The tremor is typically high frequency, in the range of 4 to 8 Hz

b. The tremor is typically most prominent with assumption of specific postures and with action

c. Alcohol intake improves the tremor

d. The tremor seen in this disorder persists during sleep

e. There is typically a family history of tremor in patients with this disorder

24. What is the most appropriate long-term therapy for this patient given her medical history and occupation?

a. Propranolol

b. Clonazepam

c. Ethanol

d. Topiramate

e. Levodopa

Questions 25–27

25. A 9-year-old boy is brought to the clinic by his parents. They reported that for the past 2 years, he had multiple gestures and behaviors. They were frustrated that he would not stop them. His teachers at school complained that he was disruptive to his classmates, and many of his prior friends would no longer spend time with him. They reported frequent grunting, sniffing, and odd loud vocalizations that sounded like a chicken clucking. In addition, he had frequent eye blinking, arm flapping, and shoulder shrugging. The patient reported that he could suppress these sounds or movements, but only for a brief period, after which he would have to do them and would feel better after doing so. Besides the abnormal movements and vocalizations, his physical examination is otherwise normal. What is the most likely diagnosis in this patient?

a. Tourette’s syndrome

b. Simple motor tic

c. Simple vocal tic

d. Complex motor tic

e. Secondary tourettism

26. Which of the following statements regarding the disorder depicted in question 25 is incorrect?

a. It is more common in males

b. To make the diagnosis, symptoms have to start before the age of 18

c. Common comorbidities include attention deficit/hyperactivity disorder and obsessive–compulsive disorder

d. The symptoms commonly improve with age

e. Its pathophysiology relates to dopamine deficiency

27. Which of the following management strategies is not typically used for treatment of patients with this disorder?

a. Clonidine

b. Risperidone or other atypical antipsychotics

c. Haloperidol or other typical antipsychotics

d. Levodopa

e. Habit-reversal therapy

Questions 28–29

28. A 21-year-old woman presents with multiple abnormal movements, including a moderate-frequency rest and postural tremor and a twisting and in-turning of her right foot. Her history was also significant for depression with suicide attempt and significant anxiety. Her MRI is shown in Figure 6.2. Laboratory tests show elevated liver enzymes. What is the most likely diagnosis in this patient?

FIGURE 6.2 Axial T2-weighted MRI

a. Essential tremor

b. Tardive dystonia and tremor

c. Wilson’s disease

d. Early-onset Parkinson’s disease

e. A psychogenic disorder

29. Which of the following statements is correct regarding the disorder depicted in question 28?

a. It is autosomal dominant in inheritance

b. It results from a mutation in the gene encoding for the copper-binding protein ceruloplasmin

c. Treatment includes copper supplementation

d. Treatment includes low zinc diet

e. Treatment includes low copper diet

Questions 30–31

30. A 32-year-old woman is brought to the clinic accompanied by her mother and brother. The patient had a 7-year history of depression with psychotic features and three suicide attempts. In recent months, she had started to have abnormal movements. The mother reported that the patient was “wiggly and jerky” all the time. On examination, she had rapid dance-like movements of all extremities. She could not protrude her tongue for a sustained period of time and move her eyes on a target while keeping her head still. The patient’s psychiatrist had attributed these movements to antipsychotic therapy he had prescribed to her for psychotic depression, but the mother was concerned because the patient’s father had similar symptoms before he died in a car accident a few years earlier. What is the most likely diagnosis in this patient?

a. Tardive dyskinesia

b. Wilson’s disease

c. Psychogenic disorder

d. Huntington’s disease

e. Benign hereditary chorea

31. Which of the following statements is correct regarding the disorder depicted in question 30?

a. It is autosomal recessive in inheritance

b. It results from a mutation on chromosome 4

c. Age of onset is stable over generations

d. It results from a single-point mutation

e. Atrophy of the brain stem is the most consistent imaging finding seen

32. Which of the following statements is incorrect regarding chorea?

a. Chorea following streptococcal infection with rheumatic fever (Sydenham’s disease) may occur months after the infection or may be the sole manifestation of rheumatic fever

b. Occurrence of chorea during pregnancy (chorea gravidarum) may suggest prior history of rheumatic fever or presence of a connective tissue disorder, such as systemic lupus erythematosus

c. Chorea may be the presenting feature of antiphospholipid antibody syndrome

d. Sydenham’s disease is best treated with dopaminergic therapies, such as levodopa

e. Benign hereditary chorea results from a mutation in thyroid transcription factor

33. A 22-year-old man is brought to the clinic by his family for multiple symptoms. One year earlier, he had begun biting his lips to the point that they were macerated. He also was having forceful tongue protrusion, to the point where he could barely eat, and he had lost significant weight. He was having wavy, writhing movements as well as more rapid jerk-like movements of his extremities. His cognitive function had declined significantly, and his speech had become slow and slurred, barely intelligible. Serum cholesterol, vitamin E, and uric acid levels were normal. Retinal examination was also normal. What is the most likely diagnosis in this patient?

a. Dentatorubral-pallidoluysian atrophy

b. Neuroacanthocytosis

c. Huntington’s disease

d. Abetalipoproteinemia

e. Lesch-Nyhan syndrome

34. A 62-year-old man with diabetes, hypertension, and dyslipidemia presents to the emergency department with sudden onset of forceful, flinging movements of his right arm and involuntary jerking of his right leg. He reports that the day prior to the onset of these movements, he had transient weakness of the right arm and leg that had then resolved. A lesion in which of the following structures would explain this patient’s symptoms?

a. Ipsilateral subthalamic nucleus (STN)

b. Ipsilateral medulla

c. Contralateral STN

d. Contralateral medulla

e. Ipsilateral frontal lobe

35. A 52-year-old man with a 30-year history of schizophrenia that had been stable for 15 years since therapy with fluphenazine was initiated, presents to the clinic complaining of involuntary movements of his tongue, mouth, and jaw. On examination, he has almost constant jaw and lip movements and tongue protrusions. Which of the following statements is correct regarding this patient’s disorder?

a. He likely has an acute dystonic reaction to antipsychotics. His antipsychotic should be stopped immediately and he should be administered an anticholinergic agent

b. He likely has tardive dyskinesia. His antipsychotic should be stopped immediately and he should be administered an anticholinergic agent

c. He likely has a primary dystonia unrelated to his prior history of medication exposure

d. He likely has tardive dyskinesia and he should be treated with higher doses of fluphenazine

e. This disorder is more likely to occur with typical antipsychotics, but can occur with atypical antipsychotics as well

36. A 22-year-old man presents to the clinic on a wheelchair. At the age of 11 years, he had started having involuntary twisting posturing of his left foot that would occur while he was playing soccer. Over time, the twisting of his foot started to occur at rest and then his entire left leg became involved. Later, his trunk and other extremities became involved as well. By the age of 16, his limbs and trunk were so “contorted” that he could no longer walk without assistance, and in the prior year, had become wheelchair bound. He had two cousins affected with a similar disorder but of less severity; his sister, who was 33-years-old, had abnormal posturing of her left foot but was otherwise fine. The patient was given a trial of levodopa without benefit. What is the most likely diagnosis in this patient?

a. Dopa-responsive dystonia (Segawa’s syndrome)

b. Primary generalized dystonia

c. A segmental dystonia

d. A psychogenic disorder, because he did not respond to levodopa

e. Juvenile Parkinson’s disease

37. A 52-year-old woman presents with head tremor and involuntary eyelid movements. She reports that for more than 10 years she had felt her head would pull to the right and slightly downward, causing pain. In recent years, she had begun to have a tremor in her head all the time, and the pain in her neck had worsened. Her head is now almost constantly pulled to the right and downward, and the only way she can relieve this is by resting her head on a pillow in a very specific position. More recently, she has begun to have involuntary eyelid closure that occurs most often when it is sunny and she is driving. She has no prior exposure to dopamine antagonists and is otherwise generally healthy. Which of the following statements is correct regarding this patient’s condition?

a. Botulinum toxin therapy is contraindicated in this disorder

b. Botulinum toxin therapy is the mainstay of treatment for this disorder

c. Her involuntary eyelid closure is likely a tic

d. She has a rare form of focal dystonia

e. The tremor in her head is likely from essential tremor

38. A 13-year-old girl is brought to the clinic by her parents, who complain that whenever it is time for the girl to participate in afterschool activities such as dance class, she starts to walk in a funny way and cannot participate in the activities. Some of the girl’s teachers had suggested that she was “faking it” to get out of dance class. On examination, she has mild cogwheel rigidity of the upper extremities and slow finger tapping bilaterally, right more than left, and gait examination showed in-turning and dorsiflexion at the left ankle. Which of the following statements is incorrect?

a. This patient likely has dopa-responsive dystonia

b. This patient’s disorder results from a mutation in the enzyme guanosine triphosphate cyclohydrolase I

c. The occurrence of her symptoms in the afternoon suggests she is malingering to get out of dance class

d. This disorder is more common in females

e. This patient will likely improve with low doses of levodopa without significant risk of dyskinesias

39. A 52-year-old woman violinist presents to the clinic complaining of abnormal contractions of her fingers that occur while she is playing her violin. These began around 1 year earlier. These contractions have been interfering with her performance, and she is concerned about losing her position in the orchestra. These contractions occur only while she is playing the violin and never during other activities. On examination, no abnormalities are seen, but on a video she brings of herself playing the violin, her fourth and fifth digits are seen to contract briefly in initial extension and then flexion. Which of the following statements is incorrect regarding this patient’s condition?

a. She has a task-specific dystonia

b. Focal botulinum toxin can be effective for this disorder

c. She is at high risk of developing generalized dystonia

d. She has a primary dystonia

e. The dystonia may overflow to more proximal areas over time

40. A 42-year-old man with congenital heart disease suffers a cardiac arrest. After 20 minutes of cardiopulmonary resuscitation, pulse is regained and blood pressure is stabilized. He remains in the ICU on mechanical ventilation for several days, but is finally extubated and discharged to a rehabilitation facility. Three months later, he presents with involuntary jerks of his arms and trunk and significant difficulty walking due to a sensation that his legs are suddenly giving out. Which of the following statements is incorrect regarding this patient’s condition?

a. He has cortical myoclonus

b. He has a form of spinal segmental myoclonus

c. His gait disorder is likely resulting from involvement of his leg muscles with the myoclonus

d. An EEG employing specific electrophysiologic techniques would show a cortical discharge prior to the myoclonus

e. This is Lance-Adams syndrome that may be seen after hypoxic–ischemic brain injury

41. A 33-year-old woman presents to the clinic complaining of clicking in her ear. On examination, rhythmic contractions of her palate are seen. Which of the following statements is correct?

a. She likely has epileptic myoclonus

b. She has palatal myoclonus, and the contractions may persist during sleep in some cases

c. This disorder results from caudate atrophy

d. Inferior olive atrophy will be seen on an MRI of her brain

e. She is having auditory hallucinations and the rhythmic contractions are voluntary

42. A 62-year-old woman presents with involuntary facial contractions. When they had started 1 year earlier, there was just involuntary jerking of her eye, and people always thought she was winking at them. Later on, they began to involve her cheek and upper lip as well. There was no pain, but there was some discomfort, and the appearance bothered her. Which of the following statements regarding this patient’s condition is incorrect?

a. A structural lesion is not identified on MRI of the brain in the majority of patients

b. This patient has hemifacial spasm

c. This patient has blepharospasm

d. Botulinum toxin injection is the mainstay of therapy for this disorder

e. Vascular compression of the facial nerve may be involved in some cases

43. A 7-year-old boy is brought to the clinic by his parents for concern of seizures. He has episodes during which he flaps his arms repeatedly against his sides and then against his head, sometimes for several minutes. During these episodes, he ignores everyone around him. These episodes most often occur when he is upset, but also in the evening around the time his father gets back home from work, particularly when his father brings home ice cream or other treats. He has a history of delayed motor and language milestones, and has been recommended special classes at school. He does not have many friends and spends most of his free time playing with the same train set, assembling and disassembling it repeatedly. Which of the following statements is correct regarding the nature of this patient’s episodes?

a. He likely has seizures and he should undergo video-EEG monitoring

b. He likely has a complex motor tic disorder

c. This patient’s history is consistent with Tourette’s syndrome

d. He has paroxysmal dyskinesias

e. This patient likely has stereotypies

44. A 12-year-old boy has multiple attacks per day, characterized by dystonic posturing of his arm and leg. These episodes most often occur when he is playing with his friends, but can occur at anytime. He is an avid basketball player, but has not been participating as much in basketball lately because the attacks occur frequently during such activities, and he has fallen or dropped the ball several times during these attacks. They last about 15 to 30 seconds, and can occur multiple times a day, depending on his level of activity. They also occur if he is startled. In between episodes, he has an entirely normal examination. Which of the following statements is correct regarding this patient’s condition?

a. This disorder results from a mutation in the nicotinic acetylcholine receptor

b. This patient has paroxysmal nonkinesigenic dyskinesia

c. A good response to anticonvulsants is seen in this disorder

d. A good response to acetazolamide is seen in this disorder

e. This is an epileptic disorder

45. A 10-year-old boy is brought to the clinic by his mother for episodes of unsteadiness. The patient experiences episodes once every few days characterized by gait unsteadiness, “as if he was drunk,” associated with “jiggling eyes,” some double vision, mild slurring of speech, and a throbbing headache. These last around 4 hours on most days, but once lasted a whole day. In between episodes, he is normal. What is the most likely diagnosis in this patient?

a. Episodic ataxia type I

b. Episodic ataxia type II

c. Episodic ataxia type III

d. Episodic ataxia type IV

e. Paroxysmal nonkinesigenic dyskinesias

46. A 45-year-old woman presents to the clinic complaining of difficulty walking. She had begun to have cramps in her arms and legs many years earlier and over the prior year, had noticed that her legs were very stiff and tight. She was having trouble walking and had fallen several times, and also reported difficulty leaning forward to pick things up off the floor. She felt “tight and stiff all over.” On examination, she had significantly increased tone in the upper and lower extremities bilaterally, visible contraction of the paraspinal muscles, exaggerated lumbar lordosis, and exaggerated startle response. Which of the following statements is incorrect regarding this patient’s diagnosis?

a. Antiglutamic acid decarboxylase antibodies are present in most patients with this disorder

b. There is an association between this disorder and insulin-dependent diabetes

c. This disorder can be autoimmune or paraneoplastic, particularly in association with antiamphiphysin antibodies

d. Levodopa is a useful therapy in patients with this disorder

e. Benzodiazepines and baclofen are useful therapies for patients with this disorder

47. A mother brings her 3-month-old boy to the clinic. She reports that sometimes when he is picked up, he stiffens up for several seconds. When he is put back down, he loosens up again. This happens consistently and is very concerning to her. He also seems to be very “jumpy”: even slightly loud or unexpected noises cause him to suddenly jerk. A diagnosis of hyperekplexia is suspected. Which of the following statements is incorrect regarding hyperekplexia?

a. Some patients respond to benzodiazepines or sodium valproate

b. Mutations in the glycine receptor and presynaptic glycine transporter have been identified in familial hyperekplexia

c. Glycine is the inhibitory neurotransmitter at spinal interneurons, including Renshaw cells and Ia inhibitory interneurons

d. Hyperekplexia results from abnormal spinal Ia inhibitory interneuron reciprocal inhibition

e. Hyperekplexia is a startle-evoked epileptic seizure

48. Regarding cerebellar anatomy, which of the following statements is incorrect?

a. Purkinje cells are inhibitory; their neurotransmitter is GABA

b. The superior cerebellar peduncle carries the majority of cerebellar efferents

c. Cerebellar hemisphere lesions lead to contralateral clinical signs

d. Climbing fibers originating from the inferior olive constitute a large component of cerebellar afferents

e. Granule cells are the only cerebellar cell types that are excitatory

49. A 63-year-old man with a history of alcoholism of more than 30 years’ duration presents to the clinic complaining of walking difficulties. On examination, he has a wide-based, lurching gait. There is minimal dysmetria on finger-to-nose or heel-to-shin testing. There are no eye movement abnormalities and no nystagmus. MRI of the brain shows cerebellar atrophy, particularly in the midline. Which of the following statements is correct regarding this man’s gait disorder?

a. His history of alcoholism is unlikely to be related as only acute alcohol intoxication leads to gait ataxia

b. Chronic alcohol exposure leads to significant cerebellar hemisphere atrophy with relative sparing of midline structures

c. In an alcoholic with ataxia, the cause is most likely a sensory neuropathy because the effects of alcohol on the cerebellum do not lead to clinical signs

d. Chronic alcohol exposure predominantly leads to atrophy of midline cerebellar structures, such as the vermis

e. Thiamine deficiency leads to memory loss and eye movement abnormalities, but not ataxia

50. Which of the following statements is incorrect regarding acquired causes of cerebellar ataxia?

a. Hypothyroidism can lead to gait ataxia, and thyroid-stimulating hormone should be checked in patients with gait ataxia

b. Celiac autoantibodies should be checked in patients with gait ataxia only if there are gastrointestinal symptoms to suggest gluten intolerance

c. The chemotherapeutic agent cytarabine can lead to irreversible cerebellar ataxia

d. Mercury and bismuth both can lead to cerebellar ataxia in toxic amounts

e. Chronic phenytoin can lead to cerebellar atrophy due to Purkinje cell loss

51. A 7-year-old boy is noted to have some clumsiness while walking. Over the subsequent 2 years, his gait becomes significantly ataxic, and he is brought to the clinic. On examination, he has reduced light-touch and vibratory sensation in the lower extremities, with impaired proprioception and hypoactive reflexes, but extensor plantar responses bilaterally. He has high-arched feet and scoliosis. His gait is wide based and lurching. He has dysmetria on finger-to-nose and heel-to-shin testing. Which of the following statements is incorrect regarding this patient’s most likely diagnosis?

a. It is autosomal recessive in inheritance

b. It results from an expansion in a CAG trinucleotide repeat in the frataxin gene

c. It result from an expansion in a GAA trinucleotide repeat in the frataxin gene

d. The frataxin protein is a nuclear-encoded mitochondrial protein

e. Cardiac conduction defects and cardiomyopathy occur in patients with this disorder

52. A 9-year-old girl is brought to the clinic for concerns of gait instability. On examination, she has reduced sensation to light-touch and pinprick, reduced vibratory sensation, and impaired proprioception. Deep tendon reflexes are absent in the upper and lower extremities. She is also noted to have truncal and limb ataxia and choreoathetosis. On cranial nerve examination, she cannot move her eyes without thrusting her head in the direction of attempted gaze. Examination also shows multiple dilated tufts of capillary loops in the conjunctiva and oral mucosa. Which of the following statements is correct regarding this patient’s most likely diagnosis?

a. It is autosomal dominant in inheritance

b. It results from a trinucleotide repeat expansion

c. It results in impaired DNA repair

d. Patients with this disorder have hypergammaglobulinemia

e. A reduced risk of malignancy is seen with this disorder

53. Which of the following statements is incorrect regarding the spinocerebellar ataxias?

a. They typically present in the third to fifth decades of life

b. They are autosomal dominant in inheritance

c. Cerebellar atrophy is seen on MRI of the brain

d. Neuropathy, upper motor neuron findings, and cognitive decline may occur

e. They all result from a CAG repeat expansion

54. A 63-year-old previously healthy man presents with tremor in both hands as well as gait instability. On examination, he has a bilateral rest and postural tremor, rigidity, bradykinesia, and a wide-based gait. Family history is remarkable only for mental retardation in his grandson. A T2-weighted image from his MRI is shown in Figure 6.3. Which of the following statements is incorrect regarding this patient’s disorder?

FIGURE 6.3 Axial T2-weighted MRI (Courtesy of Dr. Ilia Itin)

a. This disorder occurs only in males

b. MRI of the brain may show T2 hyperintensities in the inferior cerebellar peduncle and cerebellum

c. Dysautonomia may occur as part of this syndrome

d. This disorder is X-linked

e. This patient should be checked for a premutation in the fragile X gene

55. A 19-year-old woman is brought to the clinic for progressive gait disorder. She had a history remarkable for bilateral cataracts, cognitive decline, and personality changes that are of unclear etiology and under investigation. On examination, she has dysmetria, a wide-based gait, and evidence of neuropathy. Bilateral masses are noted in the Achilles tendons. Which of the following tests would be most helpful in making a diagnosis in this patient?

a. Thyroid-stimulating hormone

b. Analysis of CAG repeat number on chromosome 14

c. Serum cholesterol levels

d. Serum cholestanol levels

e. Serum copper and ceruloplasmin

56. A 72-year-old woman is brought to the clinic by her family for trouble walking. The patient reports that when she stands to walk, she feels her legs are very unsteady and she is scared of falling. Because of this fear, she prefers to get around on a wheelchair. However, if she is made to stand and finally walk, once she gets moving, her gait is fine and she has no difficulties. On examination, while seated, no tremor, rigidity, or bradykinesia are present. On standing, a high-frequency tremor in the thighs and “bobbing” of the knees is seen, and the patient expresses a sense of unsteadiness and fear of falling. While leaning on the examiner, she begins to take steps and her gait is fine and she is no longer apprehensive. Which of the following is the most likely diagnosis?

a. Normal-pressure hydrocephalus

b. Psychogenic gait disorder

c. Essential tremor

d. Vascular parkinsonism

e. Orthostatic tremor

57. A 72-year-old man is brought to the clinic by his family for slowness of movement of several years duration. On examination, he has moderate dysarthria, bilateral rigidity, and bradykinesia. Tremor is absent. An image from his brain CT scan is shown in Figure 6.4. Which of the following statements regarding this patients disorder is incorrect?

FIGURE 6.4 Axial noncontrast CT scan

a. He has bilateral basal ganglia and cerebellar hemorrhages, leading to his parkinsonism

b. The genetic disorder that can lead to these CT scan findings may be autosomal dominant or recessive

c. Hyperparathyroidism can lead to this finding

d. Hypoparathyroidism can lead to this finding

e. Parkinsonism, cerebellar signs, and dysarthria are the most common clinical findings seen

Questions 58–59

58. A 32-year-old woman presents to the clinic to request testing for Huntington’s disease. Her mother had Huntington’s disease, and she had taken care of her until her death 2 years earlier. She wants to know what her chances are of developing Huntington’s disease, so she can plan ahead for herself and her family. Which of the following statements is correct regarding management of this case?

a. The patient should not be tested because the result could have implications regarding her insurance coverage

b. The patient should be asked to prepare an advanced directive prior to genetic testing

c. Genetic counseling should be offered only if the test is positive

d. Genetic counseling should be offered to the patient both before the test and after the result is available

e. If the patient’s genetic test is positive, it is the physician’s obligation to inform her boyfriend because if they have children, the children are at risk of having the disorder as well

59. The parents of a 7-year-old boy present with him to the clinic, requesting that he be tested for Huntington’s disease. A family history of Huntington’s disease is present, and the parents would like him to be tested so that they know what to expect for him in the future and to be able to plan accordingly. He has no neuropsychiatric signs or symptoms and no evidence of chorea or other abnormal movements. What is the most appropriate next step?

a. Order genetic testing for the boy, but advise the parents that the family should receive genetic counseling if the test is positive

b. Advise the family that genetic testing will not be ordered until he is 18 years of age and makes the decision himself to be tested

c. Alert social services that the parents are putting their son’s autonomy at risk

d. Order genetic testing, but tell the parents that the results would not be revealed until their son is 18 years of age

e. Order genetic testing, but insist that the parents be tested as well

Answer Key

1. d

2. c

3. e

4. e

5. d

6. a

7. a

8. c

9. d

10. a

11. c

12. a

13. d

14. b

15. b

16. c

17. c

18. b

19. b

20. e

21. d

22. b

23. d

24. d

25. a

26. e

27. d

28. c

29. e

30. d

31. b

32. d

33. b

34. c

35. e

36. b

37. b

38. c

39. c

40. b

41. b

42. c

43. e

44. c

45. b

46. d

47. e

48. c

49. d

50. b

51. b

52. c

53. e

54. a

55. d

56. e

57. a

58. d

59. b

Answers

1. d, 2. c

The major outflow of the basal ganglia arises in the globus pallidus interna (GPi). Projections from the GPi to the thalamus are inhibitory, as discussed further below.

Projections to the basal ganglia arise from several areas. The majority of cortico-basal ganglia projections target the striatum, which consists of the caudate and putamen. The cortex also projects to the subthalamic nucleus (STN). The striatum also receives projections from the thalamus, substantia nigra, and brainstem (including projections from the norepinephrine-containing neurons in the locus coeruleus and serotonergic raphe nuclei). The lenticular nucleus (also known as the lentiform nucleus) consists of the putamen and globus pallidus. The globus pallidus is divided into globus pallidus externa (GPe) and globus pallidus interna (GPi). The major efferent pathways from the basal ganglia arise from the GPi as well as the substantia nigra pars reticulata (SNr) and project to the ventrolateral and ventroanterior nuclei of the thalamus. This projection of the GPi and SNr is inhibitory. The thalamus then projects to the motor and premotor cortex. The circuitry often referred to in discussion of the basal ganglia can therefore be summarized as cortico-striato-thalamo-cortical. Through various efferent and afferent pathways, the basal ganglia are involved in movement of the trunk and extremities and extraocular muscles, as well as in cognition and emotion.

Basal ganglia circuitry is complex, but certain simplifications allow for a basic understanding of basal ganglia function. The circuits of the basal ganglia can be thought of as consisting of two pathways: the indirect pathway and the direct pathway (Figure 6.5).

FIGURE 6.5 Schematic of normal basal ganglia circuitry: (a) direct and (b) indirect pathways (Illustration by David Schumick, BS, CMI. Reprinted with permission of the Cleveland Clinic Center for Medical Art & Photography. © 2010. All Rights Reserved)

The direct pathway (Figure 6.5a) consists of the caudate/putamen, GPi, SNr, and thalamus. In the direct pathway, the cerebral cortex sends excitatory projections to the caudate/putamen, which send inhibitory projections to the GPi and SNr. The GPi and SNr normally send inhibitory projections to the thalamus, and inhibition of the GPi and SNr therefore disinhibits the thalamus, increasing thalamic outflow to the cortex.

The indirect pathway consists of the caudate/putamen, GPe, STN, GPi, and thalamus (Figure 6.5b). In the indirect pathway, the cortex sends excitatory projections to the caudate/putamen, which sends inhibitory projections to the GPe. The GPe normally has inhibitory projections to the STN, and inhibition of the GPe by striatal projections disinhibits the STN. The STN then sends excitatory projections to the GPi. Recall that the GPi sends inhibitory projections to the thalamus. Therefore, the indirect pathway ultimately inhibits the thalamus, reducing thalamic outflow to the cortex.

The substantia nigra is a structure located between the crus cerebri and the tegmentum of the midbrain. It is a pigmented structure containing a high density of dopaminergic neurons. The substantia nigra pars compacta projects to the striatum, inhibiting the indirect pathway and exciting the direct pathway.

A mnemonic for basal ganglia circuitry is as follows:

Direct pathway: Cinnamon candy great in taste (CCGiT): Cortex → Caudate/putamen → Globus pallidus interna → Thalamus

Indirect pathway: Cinnamon candy great excitement and supergreat in taste (CCGeSGiT): Cortex → Caudate/putamen → Globus pallidus externa → Subthalamic nucleus → Globus pallidus interna → Thalamus

In summary, because projections from the GPi to the thalamus are inhibitory:

– The direct pathway is excitatory: through inhibition of the GPi increases thalamocortical activity.

– The indirect pathway is inhibitory: through excitation of the GPi decreases thalamocortical activity.

These circuits are further discussed in questions 3 and 4.

Bradley WG, Daroff RB, Fenichel GM, et al. Neurology in Clinical Practice, 5th ed. Philadelphia, PA: Elsevier; 2008.

Fahn S, Jankovic J. Principles and Practice of Movement Disorders. Philadelphia, PA: Elsevier; 2007.

3. e, 4. e

Hyperkinetic movement disorders result from reduced activity in the indirect pathway. Thalamic projections to the motor cortex are excitatory; increased activity in thalamocortical projections leads to hyperkinetic movement disorders and decreased activity in hypokinetic movement disorders.

The anatomy of the direct and indirect pathways is described in questions 1 and 2 and Figure 6.5. Because activity in the direct pathway increases activity in thalamic projections to the motor cortex, the direct pathway plays a major role in initiating and maintaining movement. A reduction in activity of the direct pathway therefore reduces movement. Similarly, because the indirect pathway decreases activity in thalamocortical projections, the indirect pathway suppresses movement. Many movement disorders can be explained by reduced or increased activity in the direct or indirect pathway. In general, hyperkinetic movement disorders (those associated with increased and/or extraneous movements) result from reduced activity in the indirect pathway, whereas hypokinetic movement disorders (those marked by a reduction in movement) result from reduced activity in the direct pathway and this will be the premise for the following discussion. (Note however that many movement disorders lead to a combination of hyperkinesia and bradykinesia, with the prototype being Parkinson’s disease (PD) (discussed in questions 6 and 7), suggesting that the indirect–direct pathway model is an oversimplification and does not fully explain many movement disorders.)

The substantia nigra pars compacta (SNc) inhibits the indirect pathway and excites the direct pathway. The SNc degenerates in PD. Reduced SNc activity leads to a reduction in thalamocortical output due to reduced excitation of the direct pathway and reduced inhibition of the indirect pathway (Figure 6.6a). Therefore, one hallmark of PD is bradykinesia. Reduced direct pathway activity obviously does not explain the tremor that occurs in PD.

FIGURE 6.6 Schematic of basal ganglia circuitry in (a) Parkinson’s disease and (b) chorea (Illustration by David Schumick, BS, CMI. Reprinted with permission of the Cleveland Clinic Center for Medical Art & Photography. © 2010. All Rights Reserved)

In disorders in which chorea is the most prominent feature, such as Huntington’s disease, a reduction of activity in the indirect pathway disinhibits the thalamus, leading to increased activity (Figure 6.6b). Bradykinesia is also a feature of Huntington’s disease, and a reduction in indirect pathway activity does not fully explain the abnormalities of movement seen in this disorder.

Bradley WG, Daroff RB, Fenichel GM, et al. Neurology in Clinical Practice, 5th ed. Philadelphia, PA: Elsevier; 2008.

Fahn S, Jankovic J. Principles and Practice of Movement Disorders. Philadelphia, PA: Elsevier; 2007.

5. d

D1 receptors are primarily found on neurons involved in the direct pathway, and D2 receptors on neurons involved in the indirect pathway.

Glutamate is the major excitatory neurotransmitter in the basal ganglia; it is the neurotransmitter released at the majority of excitatory synapses, including cortical projections to the striatum, projections of the subthalamic nucleus to the globus pallidus interna (GPi), and projections of the thalamus to the cortex. GABA is the major inhibitory neurotransmitter of the basal ganglia and is the neurotransmitter released in projections of the striatum to the globus pallidus and the GPi to the thalamus. Various peptides colocalize with these neurotransmitters, including neuropeptide Y, dynorphin, and enkephalin.

Whether or not dopamine acts as an excitatory or inhibitory neurotransmitter depends on the receptor it is acting at. There are two families of dopamine receptors: D1 and D2. The D1 to D5 subtypes of the D1 family of dopamine receptors are metabotropic receptors coupled to adenylate cyclase. D1 receptors are primarily found on neurons involved in the direct pathway, and D2 receptors on neurons involved in the indirect pathway. The substantia nigra pars compacta inhibits the indirect pathway via activity at D2 receptors in the striatum and excites the direct pathway via activity at D1 receptors in the striatum (Figure 6.5). Both D1 and D2 receptors are found in neurons of the cortex and the limbic system. D2 receptors are found in the pituitary gland; activity at D2 receptors in the pituitary inhibits prolactin release, hence the hyperprolactinemia seen in patients taking antipsychotics (which antagonize D2 receptors).

Bradley WG, Daroff RB, Fenichel GM, et al. Neurology in Clinical Practice, 5th ed. Philadelphia, PA: Elsevier; 2008.

Fahn S, Jankovic J. Principles and Practice of Movement Disorders. Philadelphia, PA: Elsevier; 2007.

Strange PG. Dopamine receptors in the basal ganglia: Relevance to Parkinson’s disease. Mov Disord. 1993; 8(3):263–270.

6. a, 7. a

Both the patients depicted in questions 6 and 7 have histories consistent with idiopathic Parkinson’s disease (PD). Parkinsonism is a general term used for patients with tremor, bradykinesia, and/or rigidity. Idiopathic PD is a disorder characterized by tremor, bradykinesia (slowness of movement), and/or rigidity, but in the setting of specific clinical features and a temporal relation of the symptoms, in the absence of other historical and clinical findings that would suggest alternative causes of parkinsonism. The United Kingdom Parkinson’s Disease Society Brain Bank’s criteria for the diagnosis of PD include the presence of bradykinesia with either rest tremor, rigidity, or postural instability, in addition to at least three of the following features: unilateral onset, rest tremor, progression over time, persistent asymmetry, response to levodopa, levodopa-induced dyskinesias, and/or a clinical course of 10 or more years. As is evident from these criteria, tremor is not necessary for the diagnosis; idiopathic PD can be diagnosed without tremor, as in the case depicted in question 7. Idiopathic PD can be further classified into tremor-predominant and akinetic-rigid forms. Other features of PD are micrographia (small handwriting), hypophonia (reduced voice volume), hypomimia (reduced facial expression), hyposmia or anosmia (reduced or absent sense of smell), and reduced blinking frequency.

The diagnosis of idiopathic PD is purely clinical. Fluorodopa PET studies in patients with idiopathic PD show reduced uptake in the putamen; this and other radiolabeled tracer imaging techniques are predominantly used in the research setting, though in some cases provide an aid in diagnosis.

Several other disorders may be marked by parkinsonism in addition to other clinical and historical features that suggest the diagnosis is not idiopathic PD. DLB is characterized by cognitive dysfunction, hallucinations, and fluctuations in mental status in addition to parkinsonism (discussed in Chapter 12). Absence of these features in the histories provided makes this diagnosis unlikely. Vascular parkinsonism is discussed in question 21, and the history provided makes idiopathic PD more likely. There is no history of exposure to antidopaminergic therapy to suggest drug-induced parkinsonism (discussed in question 21). Corticobasal ganglionic degeneration is discussed in question 20. Absence of a history of falls and no mention of extraocular movement abnormalities makes progressive supranuclear palsy unlikely (discussed in question 17).

Fahn S, Jankovic J. Principles and Practice of Movement Disorders. Philadelphia, PA: Elsevier; 2007.

8. c

Loss of postural reflexes is a feature of idiopathic Parkinson’s disease (PD), but typically occurs later in the course of the disease.

The tremor of idiopathic PD is moderate in frequency, 4 to 6 Hz, and is typically more distal than proximal. The tremor has been described as pill rolling, as if the individual is rolling a small pill between the index finger and the thumb. The tremor can involve the legs as well as the head region, most commonly the lips, chin, and jaw, and less often the neck. Although the tremor in idiopathic PD is typically a resting tremor (occurring at rest), postural tremor does occur as well. When a posture (such as outstretched arms) is assumed, there is usually a latency of a few seconds before the tremor appears (this contrasts with the postural tremor of essential tremor, in which the tremor appears immediately on assumption of a posture, as discussed in questions 22–24).

Loss of postural reflexes is usually a feature of more advanced PD, occurring several years after disease onset, often not until after 8 to 10 years. It occurs earlier in other parkinsonian disorders; for example, it occurs within the first 2 years of symptom onset in progressive supranuclear palsy (discussed in question 17) and within the first 5 years of symptom onset in multiple-system atrophy (discussed in questions 18 and 19).

Freezing, or a sudden transient inability to move, is typically a feature of advanced PD. Features of freezing include hesitation in gait initiation, sudden inability to move when walking through a narrow space, such as a door, and when a target is being approached. Rigidity, or increased resistance during passive range of motion, is a feature of idiopathic PD and is often cogwheeling, or ratchety, due to the increased tone being superimposed on a tremor. Rigidity can involve both proximal and distal axial and limb muscles. Chronic rigidity can lead to striatal hand (ulnar deviation at the wrist with flexion at the metacarpophalangeal joints and extension at the interphalangeal joints), striatal toe (extension of the big toe with flexion of the other toes), and camptocormia (extreme flexion of the spine that worsens during walking and improves in the supine position).

Fahn S, Jankovic J. Principles and Practice of Movement Disorders. Philadelphia, PA: Elsevier; 2007.

9. d

Constipation is a prominent symptom in patients with Parkinson’s disease (PD). There are many nonmotor symptoms that affect patients with idiopathic PD. In fact, nonmotor features lead to as much, if not more, disability and discomfort as do the motor features. Shoulder pain is a frequent occurrence in PD and can be present years prior to the onset of motor symptoms. Other joints can also be involved. These pains likely result from rigidity and reduced motion at joints; that these pains are due to the underlying disorder rather than arthritis or other orthopedic problems is evidenced by improvement of these pains with dopaminergic therapy. Other types of pain seen in patients with PD include that which is related to dystonia (such as dystonic foot cramping). Nonspecific sensory symptoms (such as paresthesias) in the absence of other physical examination findings indicative of neuropathy also occur.

Urinary frequency, urgency, and nocturia are common in patients with PD, occurring in both sexes and resulting from the more generalized autonomic dysfunction that occurs. In PD, urinary symptoms result in large part from detrusor hyperreflexia. Prostatic enlargement can certainly exacerbate these symptoms. Early neurogenic incontinence in a patient with parkinsonism may suggest the diagnosis of multiple-system atrophy (MSA) instead of PD. As mentioned, patients with idiopathic PD do suffer from autonomic dysfunction, including orthostatic hypotension, and the presence of orthostatic hypotension in patients with parkinsonism does not necessarily imply that their diagnosis is MSA. Unlike MSA, the orthostatic intolerance in PD is usually milder, though it may be severe, and typically occurs later in the course of the disease.

Constipation is a prominent symptom in patients with PD, and may predate the motor symptoms for years. It results from impaired gastrointestinal motility due to involvement of the enteric nervous system with the disease.

REM sleep behavioral disorder (discussed in Chapter 5) can occur in patients with various neurodegenerative disorders, predating other symptoms by years. Its occurrence does not necessarily imply the diagnosis is DLB; it can certainly occur in patients with idiopathic PD.

Fahn S, Jankovic J. Principles and Practice of Movement Disorders. Philadelphia, PA: Elsevier; 2007.

Zesiewicz TA, Sullivan KL, Arnulf I, et al. Practice parameter: treatment of nonmotor symptoms of Parkinson disease. Report of the quality standards subcommittee of the American Academy of Neurology. Neurology. 2010; 74: 924–931.

10. a

Various neuropsychiatric symptoms can occur in patients with idiopathic Parkinson’s disease (PD). Depression is common in this patient population, affecting over half of patients at some point during their disease. Depression in PD may in part result from the limitations the disorder imposes on daily life, and as a reaction to having this disease, but is also thought to be secondary to involvement of serotonergic and noradrenergic pathways in the primary disease process. Patients with PD typically have hypomimia (reduced facial expression) and apathy, but in this case, the patient’s reported symptoms suggest she is depressed. She should be treated with antidepressant medications; the choice of medications depends on various factors. The majority of antidepressants can be used in patients with PD, except for nonselective monoamine oxidase inhibitors which are contraindicated in patients being treated with concomitant levodopa because of risk of sympathetic overactivation. Anxiety and panic attacks are other common psychiatric disorders seen in patients with PD.

Fahn S, Jankovic J. Principles and Practice of Movement Disorders. Philadelphia, PA: Elsevier; 2007.

Zesiewicz TA, Sullivan KL, Arnulf I, et al. Practice parameter: treatment of nonmotor symptoms of Parkinson disease. Report of the quality standards subcommittee of the American Academy of Neurology. Neurology. 2010; 74:924–931.

11. c

This patient’s history is consistent with Parkinson’s disease (PD) with dementia. A large number of patients with idiopathic PD develop cognitive dysfunction, and dementia is not uncommon. PD with dementia is a diagnosis made when the patient meets criteria for idiopathic PD for at least 1 year before dementia onset. The dementia in these patients may be accounted for by concurrent Alzheimer’s disease in some cases; in others, the pathology relates to the presence of Lewy bodies.

In DLB (discussed further in Chapter 12), cognitive dysfunction and hallucinations antedate the parkinsonism, though they may all occur concurrently. This patient had a near-normal neuropsychologic evaluation 10 years earlier, making DLB unlikely. This patient’s history does not suggest normal-pressure hydrocephalus (NPH) (discussed in Chapter 12); NPH can lead to parkinsonism with dementia, but the history suggests this patient had typical idiopathic PD early on, and the dementia occurred years later, without mention of incontinence. Similarly, the description of the patient’s disease early on makes corticobasal ganglionic degeneration less likely (discussed in question 20). The history is not consistent with FTD (discussed in Chapter 12), though FTD and idiopathic PD can co-occur.

Jankovic J. Principles and Practice of Movement Disorders. Philadelphia, PA: Elsevier; 2007.

12. a

Parkinson’s disease (PD) results from degeneration of dopaminergic neurons in the substantia nigra pars compacta (SNc). The pathologic hallmark of idiopathic PD is the presence of Lewy bodies, which are intracytoplasmic inclusions surrounded by a clear halo (discussed further in Chapter 12). Idiopathic PD is an α-synucleinopathy. The synucleinopathies are a group of neurodegenerative disorders characterized by abnormal deposition of α-synuclein in the brain. Motor manifestations of idiopathic PD result from degeneration of dopaminergic neurons in the SNc. However, widespread pathologic changes occur in PD, including in the olfactory system, dorsal motor nucleus of the vagus nerve, locus coeruleus and raphe nuclei of the brainstem, basal ganglia, and enteric nervous system. Later, the frontal cortex becomes involved, leading to executive dysfunction. These widespread changes are thought to account for the multitude of nonmotor symptoms seen in PD.

In a minority of patients with idiopathic PD, single-gene mutations have been identified. Some of the mutated genes do not necessarily involve Lewy body deposition, pointing to the complexity of the pathophysiology of this disease; the majority of patients with PD do not have an identifiable gene mutation, further suggesting that PD is a complex disorder that is likely the result of both genetic and environmental factors. As of 2010, 15 loci on 11 genes had been implicated in familial PD; these are genotypically and phenotypically heterogeneous, and a full discussion is beyond the scope of this text, but examples include the following:

– α-synuclein (PARK1 gene), leading to abnormalities in synaptic vesicle trafficking. Autosomal dominant, young onset. Lewy body pathology is seen in this type of familial PD.

– Parkin (PARK2 gene), a ubiquitin E3 ligase. Autosomal recessive, juvenile onset. Lewy body pathology is not seen in this type of familial PD.

– Leucine-rich repeat kinase 2 (LRRK2 and PARK8 gene). The LRRK2 mutation is one of the most common causes of familial PD. Autosomal dominant. Lewy body pathology is not a prominent feature in this type of familial PD; rather, nigral degeneration is prominent.

Other genes identified in familial PD encode for proteins involved in membrane trafficking, oxidative stress, mitochondrial metabolism, and lysosomal function.

Fahn S, Jankovic J. Principles and Practice of Movement Disorders. Philadelphia, PA: Elsevier; 2007.

Hatano T, Kubo S, Sato S, et al. Pathogenesis of familial Parkinson’s disease: new insights based on monogenic forms of Parkinson’s disease. J Neurochem. 2009; 11:1075–1093.

13. d

There are several different classes of therapies for the treatment of Parkinson’s disease (PD). Entacapone is a catechol-O-methyltransferase (COMT) inhibitor. Dopamine is metabolized by COMT to form 3-O-methyldopa. COMT inhibitors inhibit this conversion, extending the plasma half-life of dopamine, prolonging its duration of action. COMT inhibitors are administered concomitantly with levodopa. They increase the duration of action of levodopa, reducing “off” periods, or periods where the patient is experiencing motor symptoms due to wearing off of the levodopa, and increasing “on” periods.

The first dopaminergic agent available for PD was levodopa, which is a dopamine precursor. After levodopa is ingested, it is converted in the brain and peripherally into dopamine by the enzyme dopa-decarboxylase (also called aromatic amino acid decarboxylase). The peripheral conversion accounts for its side effects such as nausea (discussed in question 14). Carbidopa is a peripheral dopa-decarboxylase inhibitor; it reduces conversion of levodopa into dopamine in the periphery while not inhibiting central conversion. Administration of carbidopa on its own is not of use; it is administered only in combination with levodopa.

The dopamine agonists include pramipexole and ropinirole; older dopamine agonists including bromocriptine, pergolide, and cabergoline are no longer used in the treatment of PD. Ropinirole and pramipexole are agonists predominantly at D2 and D3 receptors.

Rasagiline and selegiline are monoamine oxidase type B (MAOB) inhibitors; MAOB is involved in dopamine metabolism. The relatively selective inhibition of MAOB reduces the risk of the “cheese effect” that could be seen with concomitant intake of high levels of tyramine in certain cheeses, resulting in hypertensive crisis (the older MAO inhibitors that inhibited both MAOA and MAOB had a higher risk of this adverse effect).

Trihexyphenidyl is an anticholinergic. Its use in PD is limited to the treatment of tremor, as some of the tremor in PD is thought to result from a relative excess of acetylcholine.

Another therapy used in PD includes amantadine, which has anti-glutamatergic effects, increases presynaptic dopamine release, and inhibits reuptake of synaptic dopamine.

Fahn S, Jankovic J. Principles and Practice of Movement Disorders. Philadelphia, PA: Elsevier; 2007.

14. b

Although patients with premorbid impulse control problems are at higher risk of having worsening with dopamine agonist therapy, the dopamine agonists can cause impulse control disorders even in patients without prior history of such problems. The side effects of dopamine agonists include sedation, lower extremity edema, and impulse control problems, such as hypersexuality, pathologic gambling, and pathologic shopping, in addition to other similar behaviors. The occurrence of such impulse control problems often necessitates discontinuation of dopamine agonists.

Anticholinergic agents such as trihexyphenidyl and benztropine can cause significant cognitive dysfunction, particularly in older adults. Other side effects of these agents include constipation, dry eyes and mouth, and urinary retention.

The main side effect of levodopa is nausea; administration of extra doses of carbidopa (which inhibits peripheral conversion of levodopa into dopamine without affecting its conversion centrally) reduces this side effect. Tolerance develops over time. Dyskinesias, or extraneous choreiform movements, occur with levodopa, with the incidence being related to dosage and duration of therapy. Addition of a catechol-O-methyltransferase inhibitor increases the time dopamine is available at the postsynaptic membrane, increasing the occurrence of peak-dose dyskinesias.

Selegiline, a monoamine oxidase type B (MAOB) inhibitor, is metabolized to methamphetamine, which can lead to insomnia. The other MAOB inhibitor, rasagiline, does not have amphetamine-like metabolites.

Fahn S, Jankovic J. Principles and Practice of Movement Disorders. Philadelphia, PA: Elsevier; 2007.

15. b

In younger patients with idiopathic Parkinson’s disease (PD), there are several treatment options. In young patients with predominantly tremor, medications such as rasagiline, amantadine, or even an anticholinergic such as trihexyphenidyl can be used. Dopamine agonists can provide significant motor control, and delaying levodopa therapy may reduce risk of levodopa-induced dyskinesias.

In older adults, particularly those with cognitive dysfunction and/or hallucinations, treatment options are more limited. Given this patient’s age, and the history of daytime hypersomnolence, a dopamine agonist (such as ropinirole or pramipexole) would not be appropriate given the adverse events discussed in question 14. Bromocriptine is not used in treatment of PD any longer. Trihexyphenidyl would worsen cognitive dysfunction and do little besides perhaps improve the tremor. The most appropriate therapy for this patient would be carbidopa-levodopa.

Fahn S, Jankovic J. Principles and Practice of Movement Disorders. Philadelphia, PA: Elsevier; 2007.

16. c

Deep brain stimulation (DBS) for idiopathic Parkinson’s disease (PD) is effective in reducing tremor and bradykinesia, but not gait freezing, falls, or other axial symptoms. In patients with a measurable response to dopaminergic therapy but who over time have less benefits or significant side effects such as dyskinesias, DBS for PD has been proven safe and effective with appropriate case selection.

DBS involves the placement of electrodes into specific target areas, with the assistance of stereotaxis and intraoperative neurophysiologic recording. If symptoms predominantly affect only one side of the body, unilateral DBS to a contralateral brain target is appropriate. In patients with significant symptoms bilaterally, bilateral DBS is done. Target sites have included the subthalamic nucleus, globus pallidus interna, and ventral intermediate (VIM) nucleus of the thalamus. VIM nucleus DBS is particularly effective for tremor and is thus used for the treatment of tremor-predominant PD and essential tremor. Significant cognitive impairment is a contraindication to DBS as significant worsening of cognitive dysfunction can occur postoperatively.

Fahn S, Jankovic J. Principles and Practice of Movement Disorders. Philadelphia, PA: Elsevier; 2007.

17. c

This patient’s history and examination is consistent with progressive supranuclear palsy (PSP) (previously known as Steele-Richardson-Olszewski syndrome), a parkinsonism-plus syndrome. PSP typically has onset in the seventh decade of life, later than idiopathic Parkinson’s disease (PD) and multiple-system atrophy (MSA). The most prominent feature of this disorder is gait and balance problems, with frequent falls. Eye abnormalities in PSP include restricted vertical gaze, predominantly downward gaze, making going downstairs difficult, particularly when combined with the involuntary neck hyperextension (retrocollis) that occurs. The downward gaze restriction can be overcome by the oculocephalic maneuver. Examination often reveals square-wave jerks (jerk nystagmus in primary gaze) and impaired optokinetic nystagmus. Other parkinsonian features including bradykinesia, rigidity, micrographia, and gait freezing also occur. MRI of the brain in PSP may show atrophy of the midbrain, leading to the so-called hummingbird sign.

Unlike in idiopathic PD, falling in PSP occurs within 1.5 years of symptom onset, whereas in idiopathic PD, it typically occurs years after symptom onset. Also, the prominent vertical gaze abnormalities help distinguish PSP from idiopathic PD, though eye movement abnormalities can occur in the latter. Also unlike in idiopathic PD, the symptoms in PSP are typically less asymmetric, and response to levodopa is poor. Absence of autonomic features and/or ataxia distinguishes PSP from MSA. In addition, patients with MSA typically have forward neck flexion (antecollis), as opposed to patients with PSP who have retrocollis. PSP is a neurodegenerative disorder, and is a tauopathy, marked by abnormal deposition of the protein tau in various brain regions. In primary autonomic failure (discussed in Chapter 10), parkinsonian features and extraocular movement abnormalities are absent. Corticobasal ganglionic degeneration (discussed in question 20) does not manifest with prominent extraocular movement abnormalities (though eye movement abnormalities can occur) or early frequent falls, and is prominently asymmetric, unlike the case described.

Fahn S, Jankovic J. Principles and Practice of Movement Disorders. Philadelphia, PA: Elsevier; 2007.

18. b, 19. b

Both the patients depicted in questions 18 and 19 have multiple-system atrophy (MSA), a parkinsonism-plus syndrome. MSA has been divided into three types on the basis of the most prominent features: with significant autonomic dysfunction (MSA-A, previously known as Shy-Drager syndrome or Oppenheimer’s syndrome, as depicted in question 18), with significant parkinsonism (MSA-P), and with cerebellar ataxia being the most prominent feature (MSA-C, also known as sporadic olivopontocerebellar atrophy (OPCA), depicted in question 19). However, these may occur in combination. All these disorders share in common the presence of parkinsonism that is poorly responsive to levodopa and the neuropathologic finding of glial cytoplasmic inclusions with α-synuclein; they therefore fall under the spectrum of α-synucleinopathies, along with idiopathic Parkinson’s disease (PD). MSA has an earlier age of onset as compared to progressive supranuclear palsy (PSP), typically starting in the sixth decade of life. Autonomic dysfunction in MSA manifests with orthostatic hypotension without a compensatory increase in heart rate and urinary incontinence (due to involvement of the group of anterior horn cells in the sacral cord known as Onuf’s nucleus) and impotence. A potentially fatal feature that may occur in MSA is laryngeal dystonia.

Imaging features in MSA include hypointensity of the putamen on T2-weighted MRI, a hyperintense slit-like rim around the putamen, and the “hot-cross-buns sign,” or cruciform sign: transverse and vertical hyperintensity in the pons, as shown in Figure 6.1, due to loss of pontine neurons and pontocerebellar tracts with intact corticospinal tracts.

Poor response to levodopa, early autonomic dysfunction, and prominent ataxia distinguish the different types of MSA from idiopathic PD. Prominent early falls and restricted downward gaze distinguish PSP from MSA; as mentioned in question 17, patients with PSP have hyperextension of the neck (retrocollis), whereas patients with MSA exhibit involuntary neck flexion (antecollis). In primary autonomic failure (discussed in Chapter 10), parkinsonian features are absent. Corticobasal ganglionic degeneration is discussed in question 20; the prominent dysautonomia in question 18 and ataxia in question 19 make MSA more likely.

Fahn S, Jankovic J. Principles and Practice of Movement Disorders. Philadelphia, PA: Elsevier; 2007.

20. e

This patient’s history and examination are consistent with corticobasal ganglionic degeneration (CBGD), a parkinsonism-plus syndrome that is characterized by the presence of focal limb rigidity and/or dystonia, cortical myoclonus, and cortical sensory loss (astereognosis (inability to recognize objects placed in the hand in the absence of primary sensory loss), agraphesthesia (inability to recognize numbers or letters drawn on the hand in the absence of primary sensory loss), and loss of two-point discrimination). Other features include a frontal/subcortical pattern of cognitive dysfunction, apraxia, alien limb phenomena, and parkinsonian features such as rest tremor, rigidity, and bradykinesia. CBGD is pathologically characterized by neuronal degeneration in the pre- and postcentral cortical areas, basal ganglia, and thalamus, as well as the substantia nigra. Achromatic nuclear inclusions are seen in these areas. The presence of myoclonus, higher cortical sensory loss, and dystonic posture with absence of prominent falls makes the other choices less likely.

Fahn S, Jankovic J. Principles and Practice of Movement Disorders. Philadelphia, PA: Elsevier; 2007.

21. d

Manganese toxicity, rather than magnesium toxicity, can lead to parkinsonism. As discussed previously, parkinsonism is the general term applied when bradykinesia, tremor, and/or rigidity are present in a patient, and this term does not imply a specific cause. Many secondary causes of parkinsonism exist.

In the 1920s, during the pandemic of encephalitis lethargica or von Economo encephalitis due to the influenza virus, postencephalitic parkinsonism was seen in several patients after recovery from the acute illness. It has also been seen in more recent years with West Nile virus and Japanese encephalitis virus infection. Parkinsonism may also be a feature of the prion disease CJD.

Vascular parkinsonism results most often from multiple lacunes in the basal ganglia. It classically affects the lower extremities more than the upper extremities, with rigidity, bradykinesia, postural instability, dementia, corticospinal findings, and incontinence occurring. The upper extremities are often relatively spared in this disorder, and tremor is not a prominent feature. Some patients with vascular parkinsonism respond to levodopa therapy.

Drug-induced parkinsonism is most often seen with exposure to antipsychotic agents, with typical antipsychotics such as haloperidol being more likely to cause this adverse effect as compared to atypical antipsychotics such as risperidone. This adverse effect is dose dependent and the risk increases with increasing duration of therapy, but can occur even with low-dose brief exposure.

Manganese toxicity can occur in welders and miners, in chronic liver disease, and in patients on total parenteral nutrition, among other causes. It leads to psychiatric symptoms (“manganese madness”), parkinsonism (but usually without tremor), and a typical gait disorder characterized by toe walking with elbow flexion, the so-called cock walk. On brain MRI, hyperintensity in the basal ganglia on T1-weighted images is seen. Other toxins that can lead to parkinsonism include the neurotoxin abbreviated as MPTP (which is now used to create primate animal models of Parkinson’s disease) and carbon monoxide (which leads to relatively selective toxicity to the globus pallidus interna).

Parkinsonism can be a feature of normal-pressure hydrocephalus (discussed in Chapter 12) and other causes of chronic hydrocephalus. Structural brain lesions affecting the basal ganglia, including strokes, hemorrhages, and tumors, can also lead to parkinsonism, as can paraneoplastic processes.

Bradley WG, Daroff RB, Fenichel GM, et al. Neurology in Clinical Practice, 5th ed. Philadelphia, PA: Elsevier; 2008.

Fahn S, Jankovic J. Principles and Practice of Movement Disorders. Philadelphia, PA: Elsevier; 2007.

22. b, 23. d, 24. d

This patient’s history and examination, including her family history, are consistent with essential tremor (ET). ET is characterized by a bilateral (though sometimes asymmetric) postural tremor, typically 4 to 8 Hz with or without a kinetic component (a tremor occurring with action) that may involve the limbs, head, chin, lips, tongue, and even voice. A family history of tremor is present in the majority of patients with ET; ET exhibits an autosomal dominant pattern of inheritance with high penetrance. Although there is this strong familial component, a specific gene causing ET is yet to be identified (as of 2010). The tremor of ET classically improves with alcohol intake. Similar to most movement disorders, it does not continue in sleep (though there are exceptions, discussed in question 41).

Enhanced physiologic tremor is one of the most common causes of postural tremor, but rarely causes enough disability to require treatment. It is faster than ET, 7 to 12 Hz. Both ET and enhanced physiologic tremor increase with anxiety, but in enhanced physiologic tremor, the frequency is variable and can be slowed by mass loading (increasing weight on the arm). The presence of family history also makes ET more likely than enhanced physiologic tremor. There is no mention of abnormal posturing to suggest this patient has a dystonia with a secondary tremor (dystonic tremor). Task-specific tremor is a tremor only occurring, as the name implies, with specific tasks, such as writing or playing of musical instruments. There is no such history presented in the case. Another type of tremor is rubral tremor, also known as Holmes tremor, which is a relatively low-frequency tremor typically present at rest, with posture, and with action. It results from lesions in the dentate nucleus of the cerebellum and/or the superior cerebellar peduncle, and is often seen in patients with multiple sclerosis.

Medications used for the treatment of ET include β-blockers such as propranolol and others, primidone (an anticonvulsant which is converted into phenylethylmalonamide (PEMA) and phenobarbital), antiepileptic agents including topiramate, benzodiazepines including clonazepam, and other agents including gabapentin. Often, combinations of these therapies are necessary. In pharmacotherapy-resistant cases in which the tremor is disabling, deep brain stimulation to the ventral intermediate nucleus of the thalamus can be effective. Although ethanol intake does improve the tremor in ET, it is not an appropriate long-term therapy. Levodopa is not an effective therapy for ET. For the patient depicted in question 22, topiramate is likely the best option, as propranolol will exacerbate her asthma, and given her occupation, a sedating medication such as clonazepam would be relatively contraindicated.

Fahn S, Jankovic J. Principles and Practice of Movement Disorders. Philadelphia, PA: Elsevier; 2007.

Zesiewicz TA, Elble R, Louis ED, et al. Practice parameter: therapies for essential tremor. Report of the quality standards subcommittee of the American Academy of Neurology. Neurology. 2005; 64;2008–2020.

25. a, 26. e, 27. d

This patient’s history is consistent with Tourette’s syndrome. Tourette’s syndrome is a neuropsychiatric disorder diagnosed in the setting of at least one motor and at least one phonic motor tic that occur at some time during the illness (though not necessarily within the same time period) beginning prior to age 18. A tic is a brief and intermittent stereotypic movement (motor tic) or sound (phonic or vocal tic) that is usually preceded by a premonitory sensation. Tourette’s syndrome is associated with a variety of behavioral and psychiatric disorders, including attention deficit/hyperactivity disorder (ADHD), obsessive-compulsive disorder, anxiety, depression and other mood disorders, impulse control disorders, and others. The pathophysiology of Tourette’s syndrome is complex and not fully understood, but it is thought to involve dopaminergic hyperinnervation of the ventral striatum and limbic system.

An important part of tic management is education of the patient, family, teachers, and peers. Treatment of tics is necessary only when they cause significant social or functional impairment. Nonpharmacologic treatment options include a variety of behavioral therapies, including habit reversal therapy in which the patient is trained to reenact the tics while increasing his/her own awareness of the tics and urges in addition to other components. The mainstay of therapy for Tourette’s syndrome and other tic disorders are antidopaminergic agents such as haloperidol, pimozide, and the atypical antipsychotics. Clonidine, an α-2-adrenergic agonist, is useful for the treatment of ADHD and other behavioral aspects of Tourette’s syndrome, and improves tics as well. Levodopa would not be the first line of management in Tourette’s syndrome, as it may exacerbate tics (though some of the dopamine agonists have been shown in some studies to improve tics, possibly through reduction of endogenous dopamine turnover by action on D2 autoreceptors).

As mentioned, this patient’s constellation of tics and their duration allows for the diagnosis of Tourette’s syndrome; when only motor or phonic tics are present and/or their duration is not sufficient enough to make the diagnosis of Tourette’s syndrome, the tics are further categorized on the basis of their qualities. Tics that are classified as simple motor consist of a simple isolated movement such as eye blinking or eyebrow raising. Complex motor tics on the other hand consist of coordinated sequenced movements that resemble normal movements such as truncal flexion or head shaking. Simple phonic tics include sniffing, throat clearing, grunting, or coughing. Complex phonic tics include verbalizations such as shouting obscenities (coprolalia), or repeating others (echolalia) or oneself (palilalia). Secondary tourettism is the term given to conditions in which phonic and motor tics are present but an underlying neurologic disorder accounts for the presence of tics; for example, secondary tourettism is seen in autistic spectrum disorders, static encephalopathy, neuroacanthocytosis (discussed in question 33), Huntington’s disease (discussed in questions 30 and 31), medications, and other causes. Transient tics have occurred in some patients following infections.

Fahn S, Jankovic J. Principles and Practice of Movement Disorders. Philadelphia, PA: Elsevier; 2007.

Scahill L, Erenberg G, Berlin CM Jr, et al. Contemporary assessment and pharmacotherapy of Tourette syndrome. NeuroRx. 2006; 3(2):192–206.

28. c, 29. e

This patient’s history and MRI findings are consistent with Wilson’s disease, an autosomal-recessive disorder of copper metabolism resulting from mutations of the gene encoding the copper-transporting P-type ATPase (ATP7B) on chromosome 13. This enzyme normally binds to and transports copper across membranes. A defect in this enzyme leads to inability to excrete copper from the liver into bile, leading to copper accumulation.

The presenting symptoms of the disorder can be neurologic, hepatic, or psychiatric, or a combination of these. Abnormal movements predominate the neurologic presentation, including parkinsonism, dystonia, tremor, ataxia, as well as dysarthria. The tremor may have a variety of features, but classically, it is proximal and of high amplitude, giving the appearance of “wing beating” when the arms are abducted and the elbows flexed. A characteristic grin with drooling also occurs. Psychiatric symptoms include depression, anxiety, and less commonly psychosis. The liver disease may range from mild to fulminant hepatic failure.

Laboratory findings in Wilson’s disease include reduced serum levels of the copper-binding protein ceruloplasmin and increased urinary excretion of copper. The presence of Kayser-Fleischer rings, resulting from copper deposits in Descemet’s membrane of the cornea, also aids in the diagnosis, and patients suspected of having Wilson’s disease should be referred for slit-lamp examination. MRI of the brain shows increased signal on T2-weighted images in the caudate and putamen as seen in Figure 6.2, as well as the midbrain (with sparing of the red nucleus, leading to the so-called “double panda sign” or “face of the giant panda”) and thalamus.

The treatment of Wilson’s disease includes d-penicillamine or trientine dihydrochloride, in addition to zinc supplementation (which binds copper in the gastrointestinal tract, preventing its absorption), and a low copper diet, including avoidance of nuts, chocolate, and shellfish.

Fahn S, Jankovic J. Principles and Practice of Movement Disorders. Philadelphia, PA: Elsevier; 2007.

30. d, 31. b

This patient’s history and examination suggest that she has Huntington’s disease, an autosomal dominant disorder due to expansion of the trinucleotide repeat CAG in the Huntington gene on chromosome 4. Age of onset shows anticipation (earlier age of onset in subsequent generations).

The classic clinical features include chorea, gait instability, dystonia, and multiple neuropsychiatric symptoms including depression, psychosis, cognitive dysfunction, executive dysfunction, and personality changes. Other features include motor impersistence, demonstrated by inability of the patient to sustain tongue protrusion and impaired saccades and pursuits, with unsuppressible head movements during eye movements. In some forms, particularly the juvenile form, chorea is absent and the more prominent features are myoclonus and parkinsonism, with significant rigidity (akinetic-rigid syndrome).

In Huntington’s disease, neuronal degeneration is seen in the striatum, substantia nigra, globus pallidus, and other areas. MRI of the brain shows caudate and putamen atrophy. The exact role of the Huntington protein in the pathophysiology of Huntington’s disease is unclear. Several therapies are under investigation to alter the course of the disease, but until they become available, symptomatic treatment with anti-dopaminergic agents (such as atypical antipsychotics and tetrabenazine) is the mainstay of therapy.

When taking a history and examining a patient with abnormal movements, and in the setting of prior psychiatric history and exposure to anti-dopaminergic therapy, it is important not to attribute the abnormal movements to a tardive phenomenon (discussed in question 35), especially when other features are present that may suggest that there is an underlying disorder leading to the psychiatric symptoms, as is the case in the patient presented in question 30. The prominent chorea, motor impersistence, history of similar illness in the father, and no mention of hepatic dysfunction make Wilson’s disease less likely (discussed in question 28). A psychogenic disorder is a diagnosis of exclusion and there are several features in the history to suggest Huntington’s disease. Benign hereditary chorea in adulthood is not accompanied by neuropsychiatric changes; it is discussed further in question 32.

Fahn S, Jankovic J. Principles and Practice of Movement Disorders. Philadelphia, PA: Elsevier; 2007.

32. d

Sydenham’s disease is best treated with anti-dopaminergic therapies.

Chorea is an involuntary, rapid, and abrupt irregular movement that flows from one body part to another. It may affect any body part, and when affecting the legs and trunks, it leads to a dancing-like gait. There are several causes of chorea.

Sydenham’s disease is an autoimmune disorder manifesting with chorea that is usually bilateral but often asymmetric, as well as oculomotor abnormalities and behavioral changes following infection with group A Streptococcus. Unlike other features of rheumatic fever, the chorea can present months after the infection or may be the sole manifestation of rheumatic fever. Patients will have elevated antistreptolysin antibodies and antibasal ganglia antibodies. Treatment of the acute streptococcal infection, as well as subsequent prophylaxis with penicillin in patients who develop rheumatic fever, is essential.

Chorea occurring during pregnancy may signify prior rheumatic fever or an underlying autoimmune disease such as systemic lupus erythematosus (see Chapter 16). It may also reflect an underlying antiphospholipid antibody syndrome.

Benign hereditary chorea is an autosomal dominant nonprogressive syndrome characterized predominantly by chorea, with mild gait ataxia. It results from a mutation in the thyroid transcription factor gene. Mutations in this gene also lead to a more severe childhood syndrome characterized by mental retardation, hypothyroidism, and lung disease (so-called brain-thyroid-lung syndrome).

Fahn S, Jankovic J. Principles and Practice of Movement Disorders. Philadelphia, PA: Elsevier; 2007.

33. b

This patient’s history and examination are consistent with neuroacanthocytosis. Age of onset is usually in the third to fourth decades of life. The most prominent clinical features are orolingual dystonias, such as prominent tongue-protrusion dystonia, particularly while eating, self-mutilating behavior, cognitive decline with dementia, dysarthria, ophthalmoplegia, parkinsonism, and behavioral problems. Chorea and athetosis (a slow form of chorea) also occur. Autosomal dominant, X-linked recessive, and sporadic forms have been reported, with genetic heterogeneity. The autosomal recessive form has been associated with a mutation in the chorein gene on chromosome 9. The X-linked form is known as Mcleod’s syndrome. The diagnosis of neuroacanthocytosis is made by wet blood smear or Wright-stained blood smear demonstrating acanthocytes.

A form of neuroacanthocytosis associated with abetalipoproteinemia is associated with low serum cholesterol and vitamin E malabsorption, not present in this case. Another syndrome along this spectrum, but with normal vitamin E levels, is hypoprebetalipoproteinemia, acanthocytosis, retinitis pigmentosa, and pallidal degeneration (HARP syndrome).

Dentatorubral-pallidoluysian atrophy is a neurodegenerative autosomal dominant disorder resulting from expansion of the trinucleotide repeat CAG on chromosome 12. It is more common in people of Asian descent. It typically begins in the fourth decade of life, but earlier onset forms exist. Clinical features include myoclonus, choreoathetosis (a combination of chorea and athetosis), epilepsy, dystonia, tremor, parkinsonism, and cognitive dysfunction.

Lesch-Nyhan syndrome is an X-linked recessive disorder resulting from a mutation in hypoxanthine-guanine phosphoribosyltransferase (HGPRT), leading to abnormal purine metabolism. Hyperuricemia with nephrolithiasis, neuropsychiatric symptoms, and abnormal movements including chorea, athetosis, and rigidity occur. Self-mutilation is also a feature of Lesch-Nyhan syndrome, but the normal serum uric acid and prominent tongue-protrusion dystonia make neuroacanthocytosis more likely.

Choreoathetosis is also seen in the disorders of brain iron accumulation, such as pantothenate-kinase–associated neurodegeneration (PKAN), formerly known as Hallervorden-Spatz disease. This is an autosomal recessive disorder of childhood, marked by dystonia, other abnormal movements, and cognitive decline. It results from a mutation in pantothenate kinase 2 gene. The classic MRI appearance is one of hyperintensity in the globus pallidus with a surrounding area of hypointensity, the so-called “eye of the tiger” sign.

Huntington’s disease is on the differential diagnosis of patients with a presentation as described above, but the prominent tongue dystonia and self-mutilation make neuroacanthocytosis more likely. A normal uric acid level and the prominent tongue dystonia make Lesch-Nyhan syndrome (discussed in Chapter 14) less likely; self-mutilation is seen in both of these disorders.

Bradley WG, Daroff RB, Fenichel GM, et al. Neurology in Clinical Practice, 5th ed. Philadelphia, PA: Elsevier; 2008.

Fahn S, Jankovic J. Principles and Practice of Movement Disorders. Philadelphia, PA: Elsevier; 2007.

Stevenson VL, Hardie RJ. Acanthocytosis and neurological disorders. J Neurol. 2001; 248:87–94.

34. c

This patient’s history is consistent with hemiballism, which can result from an ischemic stroke or other lesions in the contralateral subthalamic nucleus. Ballism is a hyperkinetic movement disorder characterized by forceful, flinging, high-amplitude choreiform movements. Hemiballism, or ballism involving one side of the body, can also occur with contralateral parietal or thalamic lesions. Bilateral ballism may be due to bilateral basal ganglia infarcts. Hemiballism often responds to antidopaminergic therapy.

Fahn S, Jankovic J. Principles and Practice of Movement Disorders. Philadelphia, PA: Elsevier; 2007.

35. e

This patient’s history is most consistent with tardive dyskinesia. This disorder is an iatrogenic, typically late (hence the term tardive), adverse effect of dopamine-receptor antagonists, most commonly antipsychotics, but also seen with other therapies such as metoclopramide. It is more likely to occur with typical antipsychotics such as haloperidol and fluphenazine because of their greater antagonism at D2 receptors, but can also occur with the atypical antipsychotics such as risperidone, with clozapine and quetiapine being least likely to cause tardive dyskinesia. It may occur during therapy with dopamine-receptor antagonists or even years after the medication is discontinued. The manifestations of tardive dyskinesia include oro-bucco-lingual movements (as in this case), akathisia (inner restlessness), dystonia (often of the neck but also other body parts), tremor, parkinsonism, or a combination of these.

Abrupt cessation of a dopamine-receptor antagonist after prolonged use can lead to prominent involuntary dyskinetic movements involving various regions of the body as well as akathisia. A slow taper of the offending agent is therefore recommended when tardive dyskinesia occurs. When psychosis or other indications for dopamine-receptor antagonists persist, switching to an agent with less D2 antagonism should be attempted when possible. Dopamine-depleting agents such as tetrabenazine or reserpine have been used to treat tardive dyskinesia, with the rationale being that they will reduce dopaminergic synaptic activity without causing dopamine-receptor antagonism. Levodopa, sodium valproate, and clonazepam have also been used successfully. In pharmacotherapy-refractory cases, deep brain stimulation may be effective. Anticholinergics and antihistamines can worsen tardive dyskinesia.

Tardive dyskinesia should be distinguished from an acute dystonic reaction after administration of dopamine-receptor antagonists. This reaction typically occurs within the first few days of exposure to the agent and most often involves the ocular and face muscles, leading to oculogyric crisis (forced eye deviation) and other dystonic manifestations. Treatment involves cessation of the agent and administration of anticholinergics or antihistamines, with resolution of the dystonic reaction within hours.

Fahn S, Jankovic J. Principles and Practice of Movement Disorders. Philadelphia, PA: Elsevier; 2007.

36. b

Dystonia is a sustained contraction of agonist and antagonist muscles, leading to abnormal postures with repetitive twisting movements. Dystonia is classified as focal when it involves a single body part, as in cervical dystonia. If the dystonia spreads to a contiguous body part, it is termed segmental. Generalized dystonia refers to involvement of at least two segmental regions (such as leg plus trunk) with at least one other body part involved. Multifocal dystonia is used to describe the occurrence of dystonia in two noncontiguous body parts, such as foot and hand.

Primary generalized dystonia (also known as Oppenheim’s dystonia or dystonia musculorum deformans), depicted in question 36, is an autosomal dominant disorder resulting from a mutation in the torsin A gene on chromosome 9, and referred to as DYT1 dystonia. It is more common in those of Ashkenazi Jewish descent and has relatively low penetrance. Symptoms typically begin in childhood with action-induced limb dystonia that later spreads to involve the trunk and other limbs, with generalization of the dystonia over a few years. In some patients, the dystonia remains focal.

Response to levodopa is typically poor, and a lack of response obviously does not necessarily imply a psychogenic disorder. Treatment of primary generalized dystonia includes anticholinergics, benzodiazepines, and deep brain stimulation of the globus pallidus interna.

Dopa-responsive dystonia (discussed in question 38) shows a good response to levodopa. There are no parkinsonian features to suggest juvenile Parkinson’s disease.

Other childhood-onset generalized dystonias including DYT2, DYT4, DYT6, and DYT13 occur because of various mutations. DYT11, or myoclonus-dystonia syndrome, results from a mutation in a sarcoglycan protein (though other mutated genes have been identified) and manifests with tremor, myoclonus, and dystonia typically beginning in the teenage years and associated with various psychiatric symptoms. DYT3 dystonia, or Lubag disease, is X-linked and is seen in males of Filipino descent and manifests with dystonia and parkinsonism.

Fahn S, Jankovic J. Principles and Practice of Movement Disorders. Philadelphia, PA: Elsevier; 2007.

37. b

This patient’s history is consistent with primary cervical dystonia, the most common focal dystonia, as well as blepharospasm. Cervical dystonia typically begins in adulthood and manifests with involuntary head posture, neck pain, and in some cases a tremor (dystonic tremor; essential tremor can also lead to head tremor, but the presence of dystonia in her case makes dystonic tremor the likely diagnosis). A sensory trick (geste antagoniste), such as touching the face or head or positioning the head in a specific manner against an object, may partially relieve symptoms. In a proportion of patients with focal dystonia, blepharospasm, or dystonic eyelid movements manifesting as involuntary blinking often followed later by more forceful involuntary eyelid closure occur, often worsened with driving or light exposure. Other forms of dystonia such as oromandibular dystonia (involving the mouth and lips) may occur as well. Therapies including anticholinergics, benzodiazepines, and baclofen may be helpful, but botulinum toxin therapy is the mainstay of treatment. Blepharospasm may be seen as part of Meige’s syndrome, in which oromandibular dystonia occurs as well.

Bradley WG, Daroff RB, Fenichel GM, et al. Neurology in Clinical Practice, 5th ed. Philadelphia, PA: Elsevier; 2008.

Fahn S, Jankovic J. Principles and Practice of Movement Disorders. Philadelphia, PA: Elsevier; 2007.

38. c

This patient’s history is consistent with dopa-responsive dystonia, or Segawa’s syndrome. This disorder is more common in females and the occurrence of dystonia frequently shows a diurnal variation, being worse in the afternoon and evening. Parkinsonism is a common associated finding on examination. It typically presents in childhood, but adult-onset forms exist as well. Response to low-dose levodopa is typically present, without risk of significant dyskinesias.

Dopa-responsive dystonia, or DYT5, is autosomal dominant and most commonly results from a mutation in the enzyme GTP cyclohydrolase I (GCH1) on chromosome 14. GCH1 is the rate-limiting enzyme in tetrahydrobiopterin synthesis, which is a cofactor for tyrosine hydroxylase, the enzyme that catalyzes the rate-limiting step of dopamine synthesis.

Tyrosine hydroxylase deficiency can lead to a phenotype of dopa-responsive dystonia, but is a more severe childhood dystonia syndrome. Mutations in other genes encoding other enzymes involved in dopamine synthesis or metabolism can also lead to dystonia and other abnormal movements. Mutations in the enzyme aromatic acid decarboxylase lead to a syndrome of dystonia, parkinsonism, oculogyric crisis, dysautonomia, and other manifestations.

Bradley WG, Daroff RB, Fenichel GM, et al. Neurology in Clinical Practice, 5th ed. Philadelphia, PA: Elsevier; 2008.

Fahn S, Jankovic J. Principles and Practice of Movement Disorders. Philadelphia, PA: Elsevier; 2007.

39. c

This patient has a primary, focal, task-specific dystonia. Although the dystonia may overflow to involve more proximal areas over time during specific activities, it is unlikely that she will develop a generalized dystonia given her age.

The most common task-specific dystonia is writer’s cramp, a dystonia occurring during writing. This type of dystonia, occurring only with specific activities, is most often primary, without an underlying secondary cause. It most often occurs after the activity is performed for some time. This type of dystonia can occur during playing of various musical instruments; in horn or woodwind players, embouchure dystonia, or dystonia of the lips, jaw, or tongue can be seen. Treatment may include focal injections of botulinum toxin in a manner that minimizes the dystonia, but also minimizes impact on musical performance.

Fahn S, Jankovic J. Principles and Practice of Movement Disorders. Philadelphia, PA: Elsevier; 2007.

40. b

The following discussion of myoclonus is limited to nonepileptic myoclonus (see Chapter 5 for discussion of epileptic myoclonus). Myoclonus is a brief, sudden, jerky movement that may be either generalized (involving the whole body at once in a single jerk), multifocal (involving different body parts, not necessarily all at once), segmental or focal (involving only one region of the body).

Cortical myoclonus is the term given to myoclonus resulting from abnormal activity in the sensorimotor cortex. A cortical discharge preceding this type of myoclonus can be detected by electrophysiologic techniques that involve back-averaging. Lance-Adams syndrome, depicted in question 40, manifests after hypoxic–ischemic brain injury, and may not be evident for months or even years after the insult. The most prominent feature is an action myoclonus, and when the myoclonus involves the legs, gait is prominently affected. Treatment includes benzodiazepines such as clonazepam, piracetam, levetiracetam, as well as sodium valproate.

Myoclonus may also have brainstem origin, as well as spinal cord origin, the latter resulting in either segmental myoclonus (restricted to a specific limb or area of the trunk) or propriospinal myoclonus (involving axial muscles). Nerve root or peripheral nerve lesions can rarely result in segmental myoclonus as well. Multifocal myoclonus is often seen in the context of a variety of metabolic disorders such as uremia and liver failure; in the latter, negative myoclonus or asterixis is seen (discussed in Chapter 16). Not all myoclonus is pathologic; physiologic myoclonus includes hypnic jerks, which are hypnagogic lower extremity jerks (occurring in the early stages of sleep) and hiccups.

Bradley WG, Daroff RB, Fenichel GM, et al. Neurology in Clinical Practice, 5th ed. Philadelphia, PA: Elsevier; 2008.

Fahn S, Jankovic J. Principles and Practice of Movement Disorders. Philadelphia, PA: Elsevier; 2007.

41. b

Palatal myoclonus or tremor, depicted in question 41, is characterized by rhythmic palatal movements that may lead to audible clicks due to eustachian tube contraction (these are not hallucinations). The contractions may also involve other head regions. Palatal myoclonus may be essential (without a discernible cause) or symptomatic, due to a brainstem lesion such as stroke or tumor. The symptomatic form is one of the few movement disorders that persist during sleep. Palatal myoclonus results from dysfunction in pathways connecting the dentate nucleus of the cerebellum, the inferior olive, and the red nucleus, all of which make up the Guillain-Mollaret triangle. Hypertrophy of the inferior olive on MRI of the brain may be seen.

Bradley WG, Daroff RB, Fenichel GM, et al. Neurology in Clinical Practice, 5th ed. Philadelphia, PA: Elsevier; 2008.

Fahn S, Jankovic J. Principles and Practice of Movement Disorders. Philadelphia, PA: Elsevier; 2007.

42. c

Blepharospasm typically involves both eyes and does not involve the cheek and mouth (discussed in question 37).

This patient’s history is consistent with hemifacial spasm, in which there are synchronous contractions of one side of the face. In the majority of cases, an identifiable structural lesion is not present, but some cases can occur after facial nerve paresis or due to identifiable compressive lesions of cranial nerve VII such as a tumor or a vascular loop. Contractions most often begin around the eye and spread to ipsilateral face muscles; bilateral involvement is rare but can occur, though contractions on each side of the face are asynchronous. Treatment may involve nerve decompression if there is a clear compressive lesion, but botulinum toxin therapy is otherwise the mainstay of treatment.

Although an identifiable structural cause is usually not seen on imaging, this disorder is hypothesized to occur due to a demyelinating lesion in the facial nerve that leads to abnormal spontaneous discharges, with ephaptic transmission or spread of electrical discharges between adjacent fibers of a demyelinated nerve. Because the blink reflex test (the electrophysiologic equivalent of the corneal reflex) is abnormal in patients with hemifacial spasm, another theory is that this disorder results from hyperexcitability in the facial nerve nucleus.

Fahn S, Jankovic J. Principles and Practice of Movement Disorders. Philadelphia, PA: Elsevier; 2007.

Simpson DM, Blitzer B, Brashear A, et al. Assessment: Botulinum neurotoxin for the treatment of movement disorders (an evidence-based review): Report of the therapeutics and technology subcommittee of the American Academy of Neurology. Neurology. 2008; 70;1699–1706.

43. e

Given this patient’s history, the episodes of arm flapping are likely stereotypies. Stereotypies are patterned, repetitive, stereotyped movements, or vocalizations that occur in response to an external or internal stimulus. Common stereotypies are head nodding, arm flapping, body rocking, head banging, grunting, humming, or moaning. They may occur in otherwise normal children during times of excitement or boredom, but more often occur in children with developmental delay and autism. When associated with the latter, self-injurious stereotypies may be present, as in the case described above. In Rett’s syndrome (discussed in Chapter 14), several stereotypies are often seen, including hand wringing, body rocking, and others.

Stereotypies are distinguished from complex motor tics in that they are not associated with an urge with relief after executing the movement or vocalization. This patient’s history is not consistent with Tourette’s syndrome (discussed in questions 25-27) or paroxysmal dyskinesias (discussed in question 44).

Fahn S, Jankovic J. Principles and Practice of Movement Disorders. Philadelphia, PA: Elsevier; 2007.

44. c

This patient’s history is consistent with paroxysmal kinesigenic dyskinesias (PKDs). There are several categories of paroxysmal dyskinesias, all sharing in common episodes of hyperkinetic abnormal movements with intervening normalcy. The abnormal movements may include dystonia, chorea or choreoathetosis, ballism, or dysarthria. These disorders differ in the length of the paroxysm, triggers for the episodes, pharmacologic therapy, and genetics.

PKD is characterized by episodes that last seconds to at most 5 minutes, precipitated by sudden movement as well as by startle and hyperventilation. PKD may be either familial or sporadic, and secondary forms occur in multiple sclerosis, following trauma, in patients with a history of perinatal hypoxic encephalopathy, and in the setting of other underlying neurologic disorders. The primary form responds well to anticonvulsants such as carbamazepine.

In paroxysmal nonkinesigenic dyskinesia (PNKD), attacks last 2 minutes to several hours, and there are sometimes no clear precipitants, although episodes can be aggravated by alcohol, caffeine, and fatigue. Episodes are less frequent than in PKD. PNKD does not typically respond to anticonvulsants.

A third form of paroxysmal dyskinesias is referred to as paroxysmal exertional dyskinesias, in which episodes are triggered by prolonged exercise and last typically 5 to 30 minutes but sometimes up to 2 hours.

Paroxysmal hypnogenic dyskinesias were previously thought to be nonepileptic dyskinesias but are now known to be frontal lobe seizures as part of the syndrome autosomal dominant nocturnal frontal lobe epilepsy, which results from mutations in the nicotinic acetylcholine receptor, among other genetic mutations. The paroxysmal dyskinesias are not epileptic, but there is an association between PKD or PNKD and epilepsy; families have been reported that have both PNKD and infantile convulsions with choreoathetosis, and a mutation in the gene encoding a sodium/glucose transporter on chromosome 16 has been identified.

Fahn S, Jankovic J. Principles and Practice of Movement Disorders. Philadelphia, PA: Elsevier; 2007.

45. b

This patient’s history is consistent with episodic ataxia type II (EAII). The episodic ataxias are a group of disorders that are categorized on the basis of clinical and genetic differences. They may be familial or sporadic.

Episodic ataxia type I (EAI) is marked by episodes of ataxia in association with facial twitching that may be myokymia (rippling muscle movements) or neuromyotonia. These attacks occur up to several times per day, last seconds to minutes, and may be triggered by startle, movement, or exercise. This disorder is due to a mutation in the voltage-gated potassium channel gene KCNA1 on chromosome 12. These episodes may respond to anticonvulsants, such as carbamazepine, in some cases.

In EAII, the episodes of ataxia may be associated with brainstem symptoms such as nystagmus and dysarthria; facial twitching does not occur. The episodes last minutes to hours, can occur daily to monthly, and may be triggered by stress and alcohol intake. EAII results from a mutation in the calcium channel CACN1A4, the same gene mutated in familial hemiplegic migraine, and many patients with EAII have migraines during or outside of the ataxia attacks. EAII episodes may respond to the carbonic anhydrase inhibitor acetazolamide.

Episodic ataxia type III is an autosomal dominant disorder in which the attacks of ataxia are associated with tinnitus and vertigo, and in between attacks myokymia occurs. These attacks respond to acetazolamide. In episodic ataxia type IV, episodes of ataxia are associated with ocular motion abnormalities, and the attacks may be triggered by sudden head movement. The genes for these two types of episodic ataxia have yet to be identified (as of 2010).

Fahn S, Jankovic J. Principles and Practice of Movement Disorders. Philadelphia, PA: Elsevier; 2007.

46. d

This patient’s history and examination are consistent with stiff person syndrome. Levodopa is not useful for the treatment of this disorder.

Stiff person syndrome is a syndrome that typically begins in the fourth and fifth decades. It is characterized by increased tone affecting predominantly the axial muscles, including the paraspinal muscles, leading to exaggerated lumbar lordosis, in addition to abdominal muscles, leading to a “board-like” abdomen. The limbs later become involved. Superimposed spasms in response to anxiety or excitement occur, as does an exaggerated startle response. This disorder may be autoimmune, in association with glutamic acid decarboxylase (GAD) antibodies. GAD catalyzes the synthesis of the inhibitory neurotransmitter GABA. In patients with anti-GAD antibodies, insulin-dependent diabetes and other endocrinopathies may occur as well. More focal forms of the disorder, as in stiff leg syndrome, also occur. Stiff person syndrome may also be paraneoplastic in association with anti-amphiphysin antibodies. The mainstays of therapy for stiff person syndrome are benzodiazepines and baclofen for their GABA effect.

Fahn S, Jankovic J. Principles and Practice of Movement Disorders. Philadelphia, PA: Elsevier; 2007.

47. e

Hyperekplexia is not epileptic, but it must be distinguished from startle-evoked seizures.

Hyperekplexia, or exaggerated startle, can manifest with sudden brief exaggerated startle reactions (blinking, flexion of the neck and trunk, abduction and flexion of the arms) or more prolonged tonic startle spasms. These sometimes occur secondary to minor stimuli and do not habituate.

There are primary familial forms of hyperekplexia in which mutations in the glycine receptor and presynaptic glycine transporter have been identified. Glycine is the inhibitory neurotransmitter at spinal interneurons including Renshaw cells and Ia inhibitory interneurons, and abnormal spinal Ia inhibitory interneuron reciprocal inhibition is thought to be the cause of startle in these cases. Secondary forms of exaggerated startle occur in a variety of brainstem disorders as well as in Creutzfeldt–Jakob disease (discussed in Chapter 15) and stiff person syndrome (discussed in question 46).

Fahn S, Jankovic J. Principles and Practice of Movement Disorders. Philadelphia, PA: Elsevier; 2007.

48. c

Cerebellar hemisphere lesions lead to ipsilateral clinical signs.

The cerebellum consists of two cerebellar hemispheres, a midline vermis, and several deep gray nuclei interspersed among the cerebellar white matter. The cerebellar cortex consists of three layers. The molecular layer is outermost and consists of inhibitory neurons known as stellate and basket cells. Purkinje cells lie in a layer under these neurons, and are the main output of the cerebellum to the deep cerebellar and vestibular nuclei. The main neurotransmitter of Purkinje cells is GABA, an inhibitory neurotransmitter. The innermost layer is the granular layer, and consists of granule cells and Golgi interneurons. Parallel fibers, axons of the granule cells, travel to synapse with Purkinje cells. Granule cells are the only cerebellar cell types that are excitatory.

Inhibitory fibers arise from the Purkinje cells of the cerebellum and project to the deep cerebellar nuclei. Fibers arising from the deep cerebellar nuclei, including the dentate, emboliform, and globose nuclei, are excitatory and are carried through the superior cerebellar peduncle, decussate, and then synapse in the thalamus. The thalamus in turn projects to the cortex, which in turn projects back to the brainstem through the corticobulbar, corticospinal, and other descending pathways. Because the fibers from the cerebellum to the thalamus cross, and motor fibers from the cortex ultimately cross (in the decussation of the corticospinal tract in the pyramids), lesions to one cerebellar hemisphere lead to ipsilateral cerebellar signs and symptoms.

Afferents into the cerebellum are carried through the inferior, middle, and superior cerebellar peduncle (note that the superior cerebellar peduncle carries predominantly cerebellar efferents, though it carries afferents to the cerebellum as well). These afferents include axons of the spinocerebellar tract, which are termed mossy fibers, as well as projections from the pons, vestibular nuclei, and reticular nuclei. Another main afferent pathway to the cerebellum arises from the inferior olivary nucleus and travels in the form of climbing fibers around Purkinje cells.

The clinical features of cerebellar dysfunction include ataxia, dysmetria (uncoordinated movement with under- or overshooting of target), dysdiadochokinesia (impaired rapid alternating movements), hypometric or hypermetric saccades (under- or overshooting of eye movements to a target, respectively), hypotonia, and rebound (inability to control the extent of movement particularly when resistance is released).

Ropper AH, Samuels MA. Adams and Victor’s Principles of Neurology, 9th ed. New York: McGraw-Hill; 2009.

49. d

Chronic alcoholism can lead to a variety of neurologic signs and symptoms through several mechanisms. Malnourishment can lead to thiamine and vitamin B12 deficiency, which can lead to a variety of neurologic manifestations (discussed in Chapter 17). However, alcohol itself is toxic to the cerebellum. It predominantly affects midline structures such as the vermis, and this accounts for the prominent truncal ataxia, though cerebellar hemisphere atrophy leading to limb ataxia can also be seen.

Bradley WG, Daroff RB, Fenichel GM, et al. Neurology in Clinical Practice, 5th ed. Philadelphia, PA: Elsevier; 2008.

50. b

Celiac disease can lead to isolated cerebellar dysfunction in the absence of gastrointestinal symptoms. In a patient with ataxia of unclear etiology, celiac antibodies should be checked as gluten-free diet can improve the ataxia (discussed also in Chapter 16).

There are several causes of acquired cerebellar ataxia. Hypothyroidism can lead to gait ataxia, and checking serum thyroid-stimulating hormone is indicated in an adult presenting with ataxia. Supplementation with thyroid hormone can lead to improvement of the gait disorder. Chemotherapeutic agents including 5-fluorouracil and cytarabine can lead to significant cerebellar toxicity. In cytarabine toxicity, Purkinje cell loss and gliosis occur, and there is loss of dentate neurons as well; the cerebellar dysfunction is typically irreversible. Metals such as mercury can lead to cerebellar toxicity as well as visual cortex toxicity, leading to a syndrome of ataxia, visual field deficits, and paresthesias. Bismuth salicylate can also lead to cerebellar toxicity if ingested in high amounts. Other cerebellar toxins include the solvent toluene.

Chronic intake of phenytoin can lead to cerebellar atrophy due to damage to Purkinje cells. Acute phenytoin toxicity can lead to a reversible cerebellar ataxia. Other causes of acquired cerebellar ataxia include infection (as in HIV infection, Creutzfeldt–Jakob disease, and Whipple’s disease; see Chapter 15) or postinfection (such as is seen after varicella zoster infection in children). The Miller–Fisher variant of Guillain–Barre (discussed in Chapter 9) leads to ataxia in addition to areflexia, ophthalmoplegia, and involvement of other cranial nerves.

Bradley WG, Daroff RB, Fenichel GM, et al. Neurology in Clinical Practice, 5th ed. Philadelphia, PA: Elsevier; 2008.

Ropper AH, Samuels MA. Adams and Victor’s Principles of Neurology, 9th ed. New York: McGraw-Hill; 2009.

51. b

Friedreich’s ataxia (FA) is an autosomal recessive hereditary ataxia characterized by cerebellar dysfunction, neuropathy, and upper motor neuron findings. High-arched feet and spinal deformities occur. Cardiac involvement is common, including cardiac conduction abnormalities and hypertrophic cardiomyopathy. It most often presents in young adulthood, but presentation in early childhood or even late adulthood may occur. FA results from an expansion of the trinucleotide repeat GAA in the frataxin gene on chromosome 9. The exact role of frataxin is unclear, but it is thought to be a nuclear-encoded mitochondrial protein. Idebenone, a synthetic coenzyme Q10 analogue, improves the cardiomyopathy in FA.

Bradley WG, Daroff RB, Fenichel GM, et al. Neurology in Clinical Practice, 5th ed. Philadelphia, PA: Elsevier; 2008.

Trujillo-Martín MM, Serrano-Aguilar P, Monton-Alvarez F, et al. Effectiveness and safety of treatments for degenerative ataxias: a systematic review. Mov Disord. 2009; 24(8):1111–1124.

52. c

Ataxia–telangiectasia (AT) is an autosomal recessive disorder that typically presents in childhood with neuropathy, ataxia, and extraocular movement abnormalities, characteristically marked by inability to move the eyes without head thrusting. Telangiectasias are present in the conjunctiva and other areas. These patients are at increased risk of hematologic and other malignancies, and are prone to infections due to immunodeficiency, including hypogammaglobulinemia. This disorder results from a mutation in the ATM gene on chromosome 11. Mutations in this gene result in impaired DNA repair. A high-serum α-fetoprotein is seen in AT as well as in ataxia with oculomotor apraxia type 2, a disorder clinically similar to AT.

Bradley WG, Daroff RB, Fenichel GM, et al. Neurology in Clinical Practice, 5th ed. Philadelphia, PA: Elsevier; 2008.

53. e

The spinocerebellar ataxias (SCAs) are a group of autosomal dominant ataxias that are clinically and genetically heterogeneous. Repeat expansions are common to many of them, with the CAG repeat most often expanded, but other repeats of different lengths have been identified on different chromosomes, as have been point mutations and other genetic abnormalities. The pathophysiology of the SCAs resulting from repeat expansion is thought to relate to a toxic gain of function, leading to a protein product that is misfolded and abnormally aggregates.

The spinocerebellar ataxias (SCAs) typically present in the third to fifth decades of life, but can present at any age, with each of the SCAs having a different mean age of onset. Although clinically heterogeneous, they share in common the occurrence of a progressive truncal and limb ataxia, often with associated spasticity and other upper motor neuron findings. Other abnormalities depending on the subtype include impaired saccades and smooth pursuits, cranial nerve abnormalities, and in some cases neuropathy. In some of the SCAs, epilepsy and cognitive decline occur. In SCA7, there is retinopathy with vision loss. MRI of the brain shows cerebellar atrophy and in some cases, atrophy of the brainstem and cervical spinal cord.

The most common spinocerebellar ataxia (SCA) is SCA3, also known as Machado–Joseph disease. Like SCA1 and SCA2, age of onset is typically in the third to fourth decades of life, though wide variability again occurs. In addition to ataxia and other cerebellar signs, facial and tongue atrophy and fasciculations occur, and bulbar symptoms such as dysphagia are common. Levodopa-responsive parkinsonism may occur. Neuropathy is a late feature. SCA3 results from expansion of the CAG repeat in the gene ataxin 3 on chromosome 14.

Bradley WG, Daroff RB, Fenichel GM, et al. Neurology in Clinical Practice, 5th ed. Philadelphia, PA: Elsevier; 2008.

Ropper AH, Samuels MA. Adams and Victor’s Principles of Neurology, 9th ed. New York: McGraw-Hill; 2009.

54. a

This disorder may present clinically in females, though uncommon. This patient likely has fragile X tremor–ataxia syndrome, an X-linked ataxia. Fragile X syndrome results from expansion of CGG repeat in the FMR1 gene on chromosome X to more than 200 repeats. In the grandparents of patients with fragile X syndrome, a repeat number of 55 to 200, in the premutation range, may result in clinical manifestations, including tremor, ataxia, parkinsonism, dysautonomia, and cognitive decline. Although the clinical presentation resembles the cerebellar type of multiple-system atrophy (discussed in questions 18 and 19), a family history of mental retardation should prompt consideration of this disorder. Woman may be affected as well, though less commonly and typically with less prominent features. MRI of the brain may show hyperintensities in the cerebellum and inferior cerebellar peduncle on T2-weighted images, as seen in Figure 6.3.

Bradley WG, Daroff RB, Fenichel GM, et al. Neurology in Clinical Practice, 5th ed. Philadelphia, PA: Elsevier; 2008.

55. d

Cerebrotendinous xanthomatosis is the most likely diagnosis in this patient. This is an autosomal recessive disorder caused by a defect in the enzyme 27-sterol hydroxylase on chromosome 2, which results in deposition of cholesterol and cholestanol in a variety of tissues, including the brain, lungs, lens of the eye, and tendons. This results in a variety of clinical manifestations in multiple organ systems, including neuropsychiatric symptoms (cognitive decline, personality changes, and psychiatric symptoms), ataxia (in the limbs and trunk), parkinsonism, neuropathy, tendon xanthomas particularly in the Achilles tendon, diarrhea, and cataracts. Diagnosis is made by measurement of serum cholestanol; serum cholesterol is often not elevated and not helpful in making the diagnosis. MRI of the brain shows cortical and cerebellar atrophy and white matter abnormalities. Treatment includes chenodeoxycholic acid as a means of lowering serum cholestanol; long-term therapy may lead to improvement in neurologic signs and symptoms.

Although the tests listed would help evaluate for other disorders that can lead to ataxia and neuropsychiatric symptoms, the constellation of signs and symptoms including ataxia, cataracts, and tendon xanthomas make cerebrotendinous xanthomatosis the most likely diagnosis. Machado–Joseph disease (SCA3, discussed in question 53) is diagnosed by analysis of CAG repeat number on chromosome 14. Copper and ceruloplasmin are tested for diagnosis of Wilson’s disease (discussed in questions 28 and 29).

Bradley WG, Daroff RB, Fenichel GM, et al. Neurology in Clinical Practice, 5th ed. Philadelphia, PA: Elsevier; 2008.

56. e

This patient’s history and examination are consistent with orthostatic tremor. This type of tremor affects the trunk and thighs. It is characteristically high frequency: surface EMG electrodes on the thighs would show a tremor frequency of 14 to 16 Hz. Symptoms include unsteadiness or shakiness on standing, with improvement when given physical support or with ambulation.

The gait disorder is not described as magnetic or apraxic, and there is no history of incontinence or cognitive decline to suggest normal-pressure hydrocephalus (discussed in Chapter 12). Unfortunately, patients with orthostatic tremor may be misdiagnosed with psychogenic gait disorder because the tremor may not be visible on examination, though it is most often detectable by surface EMG. There are no features of parkinsonism on examination to suggest vascular parkinsonism. Essential tremor can affect the legs, but it would not lead to the symptoms described in the case.

Bradley WG, Daroff RB, Fenichel GM, et al. Neurology in Clinical Practice, 5th ed. Philadelphia, PA: Elsevier; 2008.

57. a

This patient’s CT scan, shown in Figure 6.4, shows bilateral calcification of the basal ganglia and cerebellum, or striopallidodentate calcinosis, also known as Fahr’s disease. The differential diagnosis of hyperdense lesions on CT scan does include hemorrhage, but given the time course of this patient’s symptoms as well as the symmetry and density of the lesions on CT scan (being comparable to that of the bone), calcium deposition is more likely. Although small basal ganglia calcifications are an incidental imaging finding of little clinical significance in most patients, the extent of calcification seen on this patient’s CT scan combined with the history and examination findings suggest that this is clinically relevant and likely contributing to his parkinsonism. The distribution of calcium deposition includes most often the caudate, but also putamen, thalamus, and cerebellum, among other areas. Striopallidodentate calcinosis can be idiopathic, but can also be seen in both autosomal dominant and recessive familial forms, and a variety of metabolic disorders including secondary hyperparathyroidism as is seen in end-stage renal disease, primary hyperparathyroidism, as well as hypoparathyroidism.

Bradley WG, Daroff RB, Fenichel GM, et al. Neurology in Clinical Practice, 5th ed. Philadelphia, PA: Elsevier; 2008.

Manyam BV. What is and what is not “Fahr’s disease.” Parkinsonism Relat Disord. 2005; 11:73–80.

58. d, 59. b

Patients should receive genetic counseling prior to obtaining a genetic test in order to allow them to make an informed decision regarding whether or not to have the test. During counseling, the positive and negative predictive value of the test, meaning of false positives and false negatives, and implications of testing should be discussed. Potential consequences including insurance concerns should be addressed, but testing should not be denied by the physician because of potential problems with insurance. Rather, the patient should be provided with information regarding potential consequences and be allowed to make an informed decision. It is the physician’s obligation to maintain strict confidentiality, and the patient should be supported regardless of when he or she is going to disclose the information to family members. Although all patients with progressive neurodegenerative disorders should be encouraged to draft advanced directives early on, this should not be a prerequisite to diagnostic testing.

On the other hand, testing of asymptomatic minors should be avoided regardless of family history; genetic testing of asymptomatic children at their parents’ request should not occur, regardless of the parents’ intentions, particularly when the disorder is not treatable and there is no intervention that can be taken to prevent the disease. Testing should occur only after the age of 18, after an informed decision has been made by the patient (after genetic counseling).

Kodish E. Testing children for cancer genes: The rule of earliest onset. J Pediatr. 1999; 135(3): 390–395.

Statement of the Practice Committee Genetics Testing Task Force of the American Academy of Neurology. Practice parameter: genetic testing alert statement of the Practice Committee genetics testing Task Force of the American Academy of Neurology. Neurology. 1996;47:1343–134.



If you find an error or have any questions, please email us at admin@doctorlib.org. Thank you!